Evolve: Maternity - Women's Health/Disorders

अब Quizwiz के साथ अपने होमवर्क और परीक्षाओं को एस करें!

A nurse on the postpartum unit discusses breast care with a client who is formula feeding her newborn. Which statement indicates to the nurse that more teaching is needed?

"Applying heat to my breasts will help ease the discomfort."

After an uneventful 8-hour labor a client gives birth. Once the airway has been ensured and the neonate has been dried and wrapped in a blanket, the nurse places the newborn in the mother's arms. The mother asks, "Is my baby normal?" What is the best response by the nurse?

"Let's unwrap your baby so you can see for yourself."

By how much should the nurse instruct a pregnant client to increase her daily protein intake?

10g -Ten grams is the amount of increase in the daily protein intake recommended for pregnant women by the Food and Nutrition Board of the National Academy of Sciences. Twenty, 30, and 40 g are all more than the recommended amount, although most women in developed countries exceed the requirement.

A client at 10 weeks' gestation phones the prenatal clinic to report that she is experiencing some vaginal bleeding and abdominal cramping. The nurse arranges for her to go to the local hospital. The vaginal examination reveals that her cervix is dilated 2 cm. What diagnosis should the nurse expect?

Inevitable abortion -Once cervical dilation has begun, the abortion is classified as inevitable. In septic abortion the cervix is dilated and there is bleeding; also, the discharge is malodorous. Bleeding and cramping may be present, but the cervix is still closed in a threatened abortion. The products of conception have been partially expelled in an incomplete abortion.

A pregnant client with an infection tells the nurse that she has taken tetracycline (Tetracyn) for infections on other occasions and prefers to take it now. The nurse tells the client that tetracycline is avoided in the treatment of infections in pregnant women because it:

Influences the fetus's teeth buds -Tetracycline (Tetracyn) has an affinity for calcium; if used during tooth bud development it may cause discoloration of teeth. Tetracycline does not adversely affect breastfeeding, cause fetal allergies to the medication, or increase the fetus's tolerance of the medication.

A client arrives at the fertility clinic for a diagnostic workup and is told by the nurse to prepare for a Papanicolaou (Pap) test. The client states, "I do not want this test. I want to speak to the person in charge." How should the nurse respond to this statement?

Inform the health care provider of the client's request

After a mastectomy, a client returns from surgery with a closed suction drainage system in place and a dry sterile dressing covering the incision. What should the nurse do when observing this client for signs of bleeding?

Inspect the bedclothes under the client's axillary area for signs of drainage

A client is scheduled to have a contraction stress test (CST) to determine fetal well-being. Which type of fetal heart rate (FHR) decelerations constitutes a nonreassuring outcome?

Late -The fetus with a borderline cardiac reserve will show hypoxia, evidenced by a decreased FHR with minimal stress, making the test positive. Early decelerations, a response to head compression, are benign. "Baseline" is not used to describe the baseline measurement; the baseline rate is determined before the test or early in the test to provide a basis for comparison, not to indicate fetal compromise. Variable decelerations are nonuniform drops in FHR before, during, or after a contraction; these are related to partial, brief cord compression that can be eliminated with a change in the mother's position.

What factor identified by the nurse in a client's history places the client at increased risk for breast cancer?

Late beginning of childbearing

A 23-year-old woman comes to the clinic for a Pap smear. After the examination, the client confides that her mother died of endometrial cancer 1 year ago and says that she is afraid that she will die of the same cancer. Which risk factor stated by the client after an education session on risk factors indicates that further teaching is needed?

Late-onset menarche

A client gives birth to a baby weighing 7 lb 2 oz and decides to breastfeed. The nurse is instructing the client regarding breastfeeding. What should the nurse tell the client to expect?

Lochial flow will be increased.

A pregnant client is experiencing nausea and vomiting. The nurse determines that this discomfort:

May be related to an increased human chorionic gonadotropin level

A nurse is caring for several new mothers in the birthing unit, all in the taking-in phase of the postpartum period. What information is most appropriate for these clients at this time?

Perineal care -During the taking-in phase a woman is primarily concerned with being cared for and being cared about. Infant feeding is best taught during the taking-hold phase of postpartum adjustment. Infant hygiene is best taught during the taking-hold phase of postpartum adjustment. Family planning is not a primary concern during the immediate postpartum period.

A pregnant client asks the nurse for information about toxoplasmosis during pregnancy. What should the nurse teach the client?

Pork and beef should be cooked thoroughly.

A pregnant client asks a nurse for information about toxoplasmosis during pregnancy. What should the nurse teach the client about how to prevent the transmission of toxoplasmosis?

Pork and beef should be cooked well before being eaten.

Which action involving client needs may a nurse delegate to a nursing assistant?

Providing ice chips to a primigravida in early labor per order

A 37-year-old client with a nontender palpable breast mass has a questionable mammogram. She is undergoing further diagnostic tests to determine whether the mass is malignant. What information should the nurse take into consideration before planning health teaching for this client?

Results of a biopsy are necessary before a specific form of therapy is selected.

Rho(D) immune globulin (RhoGAM) is prescribed for an Rh-negative client who has just given birth. Before giving the medication, the nurse verifies the newborn's Rh factor and reaction to the Coombs test. Which combination of newborn Rh factor and Coombs test result confirms the need to give Rho(D) immune globulin ?

Rh positive with a negative Coombs result

Before the administration of Rho(D) immune globulin (RhoGAM) the nurse reviews the laboratory data of a pregnant client. Which blood type and Coombs test result must a pregnant woman have to receive RhoGAM after giving birth?

Rh-negative and Coombs negative -Rho(D) immune globulin (RhoGAM) is given to an Rh-negative mother after birth if the infant is Rh positive and the Coombs test reveals that the mother was not previously sensitized (negative). An Rh-positive mother will not develop antibodies to a fetus who is either Rh positive or Rh negative; therefore the Coombs test is not performed.

A nurse is planning to administer Rhogam (Rh immune globulin). Which situation requires the administration of this medication?

Rh-negative woman who has had an amniocentesis

A 36-year-old client undergoes a modified radical mastectomy. The nurse determines that the client understands the schedule for self-examination of her remaining breast when she states that she will examine her breast:

Seven days after each menstrual period

When teaching a client about using a diaphragm as a form of contraception, the nurse should tell her that the diaphragm:

Should remain in place for at least 6 hours after intercourse

A breastfeeding mother asks the nurse what she can do to ease the discomfort caused by a cracked nipple. What should the nurse instruct the client to do?

Start feedings on the unaffected breast until the affected breast heals

On the first postpartum day, a client whose infant is rooming in asks the nurse to return her baby to the nursery and bring the baby to her only at feeding times. How should the nurse respond?

"It seems that you've changed your mind about rooming in."

A client with mild preeclampsia is told that she must remain on bedrest at home. The client starts to cry and tells the nurse that she has two small children at home who need her. How should the nurse respond?

"Let's explore your available current support and opportunities for child care."

A client who is scheduled for an amniocentesis says, "I'm glad that this test will show if my baby is well." How should the nurse respond?

"Potential defects caused by chromosomal errors can be detected."

On the first day after a mastectomy, a nurse encourages the client to perform exercises such as flexion and extension of the fingers and pronation and supination of the hand. The client asks why she has to do these exercises. The best response by the nurse is:

"They will help stimulate peripheral circulation."

A client has just been informed by the health care provider that she has cervical polyps. The client asks the nurse whether she should worry about them. How should the nurse respond?

"They're usually benign, and a biopsy rules out a malignancy."

A nurse is caring for a postpartum client. Where does the nurse expect the fundus to be located if involution is progressing as expected 12 hours after birth?

1 cm above the umbilicus

A couple expresses a desire to use the rhythm method of birth control. The woman tells the nurse that she menstruates every 32 days. What should the nurse teach the couple about when the client's ovulation probably occurs?

14 days before the start of the next menses

A nurse is planning care for a client who gave birth to a preterm male infant. What most common response does the nurse anticipate that the mother may experience?

Feelings of failure and loss of control

A nurse teaches a pregnant woman about the need to increase her intake of complete proteins. Which foods identified by the client indicate that the teaching is effective? (Select all that apply.)

2 Milk 3 Eggs 6 Cheese -Milk contains animal proteins, which are complete proteins that contain all of the essential amino acids. Eggs contain animal proteins, which are complete proteins that contain all the essential amino acids. Cheese contains milk, which is a complete protein that contains of all the essential amino acids. Nuts are incomplete proteins. Bread is not a complete protein. Beans are not complete proteins unless eaten in a specific combination with soy products.

Methods of relieving back pain are explained during a childbirth class. What activities identified by the client permit the nurse to conclude that the teaching has been understood? (Select all that apply.)

2 Pelvic rocking 3 Forward tilting 4 Sacral pressure -Pelvic rocking eases tension in the muscles of the lumbar region. Lumbar pain during pregnancy results from the changes in posture as the uterus grows. Forward tilting eases tension in the muscles of the lumbar region. Lumbar pain during pregnancy results from the changes in posture as the uterus grows. Applying the heel of the hand to the laboring client's sacral area (counterpressure) helps relieve the back discomfort that may result when a fetus is in the occiput posterior position.

A client with a large fetus is to have a pudendal block during the second stage of labor. What does the nurse plan to instruct the client about the effectiveness of the block? (Select all that apply.)

2 Perineal pain will not be felt. 5 The bearing-down reflex will be diminished.

A nurse in a family planning clinic determines that a client understands the discussion about using a cervical cap with a spermicide when the client states that after intercourse, a cervical cap must be left in place for at least:

6 hours

A client seeking family planning information asks the nurse during which phase of the menstrual cycle an intrauterine device (IUD) should be inserted. Before responding the nurse recalls that the insertion usually is done:

Between the first and fourth days of the cycle

A few weeks after discharge, a postpartum client experiences mastitis and telephones for advice concerning breastfeeding. The nurse notifies the practitioner to have antibiotics prescribed. What should the nurse recommend that the client do?

Breastfeed often to keep the breasts empty.

A nurse is teaching a client about the oral contraceptive prescribed by the primary health care provider. Which condition identified by the client indicates understanding of when the drug should be stopped immediately and the health care provider notified?

Chest pain

A primigravida at 12 weeks' gestation complains of nausea and vomiting during a visit to the prenatal clinic. Which pregnancy hormone should the nurse explain is thought to be responsible for nausea and vomiting during the first trimester?

Human chorionic gonadotropin (hCG)

After treatment for a bladder Infection; a client asks whether there is anything she can do to prevent cystitis in the future. What is the best response by the nurse?

Increase daily fluid consumption.

A woman in the family planning clinic has decided to use the diaphragm for contraception. What should the nurse teach her about using a diaphragm?

Insert the diaphragm before intercourse and leave it in at least 6 hours after intercourse to kill all the sperm.

What nursing care is required for a client with a radium implant for cancer of the cervix?

Limiting the client's activity to avoid dislodging the radium insert

A client comes to the fertility clinic for hysterosalpingography using radiopaque contrast material to determine whether her fallopian tubes are patent. When preparing for the test, the nurse explains to the client that she:

May have some persistent shoulder pain for 14 hours after the test -The nurse is describing referred pain from passage of the contrast medium through the tubes; it is usually indicative of tubal patency. An anesthetic is not given; the client's complaint of pain can be managed with a position change and mild analgesics. The client may resume usual activities as soon as the test is over. The client usually does not experience nausea or vomiting.

What does the nurse teach a client to do when performing breast self-examination?

Squeeze the nipples to examine for discharge

A nurse is evaluating a client's understanding regarding postoperative concerns after mastectomy. Which development near and around the incision noted by the client should be reported to her practitioner?

Swelling with erythema

The healthcare provider of a woman who had a mastectomy has prescribed a Reach to Recovery visit. What does the nurse identify as the primary reason for the referral?

To prevent social isolation -The Reach to Recovery visit helps the client meet her need to remain within her social milieu and informs her about available community resources. Teaching arm exercises, meeting the client's physical needs, and viewing her surgical incision are all responsibilities of health care professionals.

A woman arrives at the prenatal clinic stating that her pregnancy test is positive. She asks the nurse for information about an abortion. After verifying that the woman is at 8 weeks' gestation, the nurse counsels her that having an abortion is controversial and that many women have long-lasting feelings of guilt after an abortion. What is the nurse's legal responsibility?

To provide the client with correct, unbiased information

When discussing dietary needs during pregnancy, a client tells the nurse that milk causes her to be constipated at times. What should the nurse teach the client?

Treat constipation when it occurs and continue drinking milk.

A pregnant client with iron-deficiency anemia is prescribed a daily iron supplement. What nutrient should the nurse suggest that the client include in her diet to potentiate the effect of the iron supplement?

Vitamin C -Iron absorption is pH dependent; therefore iron should be taken with a source of ascorbic acid to enhance duodenal absorption. Biotin, lecithin, and vitamin B complex are all unrelated to the absorption of iron.

What should the nurse teach a formula-feeding mother about breast engorgement when it occurs?

Wear a tightly fitted brassiere.

A client with endometriosis asks the nurse what side effects to expect from leuprolide (Lupron). What should the nurse include in the response?

Weight gain

A client is admitted in preterm labor. After intravenous tocolytic medications are administered, contractions cease and she is discharged. She is to receive oral terbutaline (Brethine) 5 mg every 6 hours at home. When should the nurse advise the client to take the medication?

With food `

A client is admitted with a diagnosis of torsion of the testes. How should the nurse respond when the client asks, "Why do I have to have surgery right now?"?

"Irreversible damage occurs after a few hours."

What nursing intervention should be implemented routinely after a client has a vacuum aspiration abortion?

Encouraging the client to take the prescribed antibiotic medication

A nurse is writing a teaching plan about osteoporosis. How should the nurse explain what osteoporosis is?

It involves a decrease in bone substance.

During the postpartum period a nurse determines that a client's rubella titer is negative. What action should the nurse plan to take?

Obtaining a prescription for immunization at discharge

A client is scheduled for a modified radical mastectomy. What nursing intervention is most important in the client's preoperative plan of care?

Allowing her to express her feelings about surgery

The nurse teaches a high school sex education class that herpes genitalis infection cannot be cured but that the disease is marked by remissions and exacerbations. What else should the students be taught about this infection?

Although exacerbations occur they are not as severe as the initial episode.

The nurse is caring for a first-time mother at her first prenatal visit. The client confides, "I'm not sure about all this." Which research-based knowledge guides a nurse regarding the emotional factors of pregnancy?

Ambivalence and anxiety about mothering are common.

On a return visit to the fertility clinic a couple requests fertility drugs because, despite having a 28-day menstrual cycle and temperature readings that demonstrate an ovulatory pattern, the woman has been unable to conceive. What should the nurse explain to the couple?

An examination of semen will be needed.

A female client came to the clinic with suspected primary syphilis. What sign of primary syphilis does the nurse expect the client to exhibit?

An indurated painless nodule on the vulva that is draining

A client had a cesarean birth 3 days ago. Where should the nurse, while palpating her fundus, expect to locate the fundus if each line represents 1 cm?

c

A client comes to the clinic for a 6-week postpartum checkup. She confides that she is experiencing exhaustion that is not relieved by sleep and feelings of failure as a mother because the infant "cries all of the time." When asked whether she has a support system, she replies that she lives alone. Which response would provide the most accurate information?

Asking the client questions, using a postpartum depression scale

On her second visit to the fertility clinic, a client whose temperature charts demonstrate an ovulatory pattern and a regular menstrual cycle requests fertility drugs. What is the nurse's best intervention?

Asking the client to have her partner's semen examined -Because the client is ovulating, the inability to conceive may be related to her partner's fertility.

A client who is to undergo dilation and curettage, and conization of the cervix for cancer appears tense and anxious. What is the best approach for the nurse to support the client emotionally?

Asking whether something is troubling the client and whether she'd like to talk about it

Which client care activity may a nurse safely delegate to a nursing assistant?

Assisting a client who is recovering from an abdominal hysterectomy to the bathroom

In the second hour after a client gives birth, her uterus is found to be firm, above the level of the umbilicus, and to the right of midline. What is the appropriate nursing intervention at this time?

Assisting the client to the bathroom to empty her bladder

A primigravida complains of morning sickness. What should the nurse plan to teach her?

Avoid long periods without food

During the discharge conference with a client who has had a hysterectomy the nurse includes instructions for avoiding the thromboembolic phenomena that may occur as a complication. What should these instructions include?

Avoid sitting for long periods of time.

A primigravida tells the nurse that she has morning sickness. What suggestion should the nurse make to help relieve the nausea?

Avoiding long periods without food

A client has a total hysterectomy with bilateral salpingo-oophorectomy for cancer of the ovary. In addition to encouraging ambulation, what exercise should the nurse instruct the client to perform to help prevent postoperative deep vein thrombosis (DVT)?

B

A nurse is counseling a pregnant client with iron-deficiency anemia about when and how to take supplemental iron. What time of day and with what drink is iron absorption most efficient?

Before breakfast with orange juice -Iron should be taken before breakfast, on an empty stomach, to permit maximal absorption; ascorbic acid enhances the absorption of iron.

How does the nurse know whether a client is in true labor?

Contractions occur every 5 to 10 minutes; the cervix is dilated 2 cm and 75% effaced, and dilation has increased to 3 cm in 2 hours.

A pregnant client at 30 weeks' gestation begins to experience contractions every 5 to 7 minutes. She is admitted with a diagnosis of preterm labor. Although the client is being given tocolytic therapy her cervix continues to dilate, and it is determined that a preterm birth is inevitable. Which medication does the nurse expect the health care provider to prescribe?

Dexamethasone -Dexamethasone is a glucocorticoid that stimulates the production of fetal lung surfactants, which are needed for fetal lung maturity; administration is started 48 hours before the expected birth. Norgestrel is a contraceptive hormone; it is not used for preterm labor. Aminophylline is a bronchodilator; it is not used for preterm labor. Magnesium sulfate is used for tocolytic therapy and has been somewhat effective in delaying preterm labor. However, this client's labor is progressing and the birth is inevitable.

A nurse is counseling a client with type 1 diabetes who has requested contraceptive information. On which method of contraception should the nurse place the most emphasis?

Diaphragm

What is the safest and most reliable birth control method for the nurse to recommend to a client with type 1 diabetes?

Diaphragm with a spermicidal gel

A client in labor is receiving an oxytocin (Pitocin) infusion. What should the nurse do first when repetitive late decelerations of the fetal heart rate are observed?

Discontinue the oxytocin infusion

A client at 16 weeks' gestation arrives at the prenatal clinic for a routine visit. During the examination the nurse notes bruises on the client's face and abdomen. There are no bruises on her legs and arms. Further assessment is required to confirm:

Domestic abuse

The nurse discusses fetal weight gain with a pregnant client. When does it usually show a marked increase?

During the third trimester --During the third trimester the fetus is laying down fat deposits and gaining the most weight. Fetal weight gain occurs throughout pregnancy, but it is most marked in the third trimester. There is little fetal weight gain during the first trimester, when organ development is occurring.

A 26-year-old woman whose sister recently had a mastectomy calls the local women's health center for an appointment for a mammogram. What should the nurse tell the client when preparing her for the mammogram?

Each breast will be firmly compressed between two plates.

A woman visits the clinic because she has dysmenorrhea. What goal should the nurse identify for this client?

Easing the pain of the client's menstruation.

A client had a fourth-degree perineal laceration during the birth of her neonate. What should the nurse recommend to protect the area from additional trauma?

Eat a high-fiber diet with increased fluid intake.

A client is receiving antibiotics and antifungal medications for the treatment of a recurring vaginal infection. What should the nurse encourage the client to do to compensate for the effect of these medications?

Eat yogurt daily

What is the priority nursing intervention during the 2 hours after a cesarean birth?

Evaluating the lochia to identify the complication of hemorrhage

A client with a benign ovarian tumor undergoes laparoscopic surgery. What should the nurse include in the postoperative teaching?

Expect shoulder pain for 12 to 24 hours.

A 47-year-old client comes to the clinic for a Papanicolaou (Pap) smear. She tells the nurse that she has been experiencing hot flashes and that her periods have been occurring at longer, less regular intervals, with a scanty flow. What does the nurse conclude is the most likely cause of these changes?

Expected menopausal changes

When discussing future health management with a client who has had a total hysterectomy, the nurse advises regular physical examinations. The client agrees and adds, "It won't be so hard to go now that I won't need the pelvic examination and Pap smear." How should the nurse respond?

Explain why regular pelvic examinations and Pap smears of vaginal secretions will be necessary in the future.

During a class for prepared childbirth, the nurse teacher discusses the importance of the spurt of energy that occurs before labor. Why is it important to conserve this energy?

Fatigue may influence pain medication requirements. -Fatigue will interfere with the successful use of other coping strategies such as distraction; this may lead to the client's need for pain medication.

A nurse is observing the electronic fetal monitor as a client in labor enters the second stage. The nurse identifies early decelerations of the fetal heart rate with a return to the baseline at the end of each contraction. What does this usually indicate?

Fetal head compression -Early decelerations are expected occurrences as the fetal head passes through the birth canal; the fetal heart rate returns to baseline quickly, indicating fetal well-being. The data do not indicate that the mother has diabetes. Variable decelerations occur with umbilical cord compression, not prolapse. Maternal hypotension will cause late decelerations because of fetal hypoxia.

A pregnant client who is scheduled for a nonstress test (NST) asks a nurse how the test can show that "my baby is all right." The nurse explains that it is a way of evaluating the condition of the fetus by comparing the fetal heart rate (FHR) with:

Fetal physical activity -The FHR should increase with physical activity; a reactive NST reveals accelerations of 15 beats/min, lasting 15 seconds with fetal movement. This response indicates fetal well-being.

The clinic nurse is providing home care instructions for a client with pelvic inflammatory disease. What resting position should be recommended by the nurse?

Fowler

A primigravida at 8 weeks' gestation is visiting the prenatal clinic for the first time. What should an examination reveal at this time?

Goodell's sign

The nurse is teaching a prenatal class to expectant mothers in their first trimester of pregnancy. In addition to discussing the need for 0.6 mg/day of folic acid replacement, which dietary choice that is high in folic acid should the nurse recommend?

Half cup of cooked spinach -A half cup of cooked spinach provides 100 mcg of folic acid per serving. One egg, a slice of bread, and half a cup of corn each provides only 20 mcg per serving.

A married 35-year-old client is to undergo a tubal ligation. The factor in the history that contributes most to the healthy resolution of any emotional problem associated with sterilization is that the client:

Has a son and daughter and feels that her family is complete

The husband of a woman who had her fourth child 3 weeks ago states she has been irritable and crying frequently since bringing her newborn home. He asks the nurse whether this is normal. The nurse tries to help him understand the situation by stating that:

Having four children is tiring and assistance may be needed.

A client with stage 4 ovarian cancer is admitted for dehydration. The client is to receive an intravenous bolus of 500 mL D5W for 1 hour, after which the rate is to be changed to 150 mL/hr. The drop factor is 15 gtt/mL. At what rate, in drops per minute, should the nurse regulate the IV after delivery of the 500-mL bolus?

38

A client's nipples become sore and tender as a result of her newborn's vigorous suckling. What should the nurse recommend that the mother do to alleviate the soreness? (Select all that apply.)

4 Expose the nipples to air several times a day. 5 Apply hydrogel pads to the nipples after each feeding.

A nurse is providing dietary counseling to a client at 14 weeks' gestation. The client is a recent immigrant from Asia, and the nurse explores the foods that the client usually eats. Which foods should the nurse counsel the client to avoid during pregnancy? (Select all that apply.)

4 Raw shellfish 5 Herbal supplements 6 Soft-scrambled eggs

A nurse is giving discharge instructions to a client who has undergone anterior colporrhaphy. The nurse knows the teaching has been understood if the client says:

" For discomfort I can try using warm compresses on my abdomen."

A mother is seeing her newborn, who has visible birth defects, for the first time. When she sees her baby, she becomes disturbed, pushes away, and tells a nurse, "Oh, take the baby away; I never want to see it again." What does the nurse conclude from this behavior?

The response is a common one in a new mother who is finding it difficult to accept that her newborn is less than perfect.

A client who has had a mastectomy asks what the term ERP-positive means. How should the nurse explain this finding?

The tumor cells generally exhibit a positive response to hormone therapy that reduces estrogen.

A pregnant client tells the nurse in the prenatal clinic that although she and her husband do not have the disease, she has a 1-year-old daughter with sickle cell anemia. She asks the nurse, "Will this baby also have sickle cell anemia?" How should the nurse respond?

"The chance that another child will have sickle cell anemia is 25%."

A client is scheduled for a laparoscopic bilateral tubal ligation. What should the nurse include in preoperative teaching?

"You will be admitted as an outpatient for same-day surgery." -A laparoscopic tubal ligation takes about 20 minutes to perform. The client is admitted as an outpatient and goes home the same day after she recovers from the anesthesia.

A client who is scheduled to have an abdominal panhysterectomy asks how the surgery will affect her periods. How should the nurse respond?

"You won't have any more periods."

The nurse is conducting teaching for a client being discharged after an abdominal hysterectomy. Which statement by the client indicates a need for further teaching?

"I'm glad I'll be able to get back into my jogging routine next week."

A nurse is instructing a client to cough and deep-breathe after an emergency cesarean birth. The client says, "Get out of here. Can't you see that I'm in pain?" Which response will be the most effective?

"If you can't cough, try taking six very deep breaths."

A new mother who has begun breastfeeding asks for assistance removing the baby from her breast. What should the nurse teach her?

"Insert your finger in the corner of the baby's mouth to break the suction."

The nurse has completed a prenatal class for women who are expecting their first babies. Which statement by a pregnant woman indicates the need for additional teaching?

"During pregnancy it's safe for me to use my regular herbal remedies."

A client has a child with Tay-Sachs disease and wants to become pregnant again. She tells the nurse, "I'm worried it will happen again." How should the nurse respond?

"Have you considered the option of genetic counseling?" -Asking the client whether she has considered genetic counseling ensures that the client is informed of the need for genetic counseling and gives her an option for decision-making.

A nurse teaches a client with asthma about her illness during pregnancy. Which statement by the client indicates that the nurse's teaching has been effective?

"I can use my albuterol inhaler if it's absolutely necessary."

A nurse is teaching a woman how to perform breast self-examination. Which statement indicates that the client needs further education?

"My breasts are so tender right before my period that I hate doing it."

A nurse is teaching a prenatal class about infant safety. After the class several of the students are heard discussing what they have learned. The nurse determines that the teaching has been effective when one of the future parents states:

"My mother can't believe that babies are supposed to sleep on their backs, not their stomachs."

A client at 16 weeks' gestation is being treated for Trichomonas vaginalis. Which statement best indicates to the nurse that the client has learned measures to prevent a recurrence?

"My partner has to get treated before we have sex again."

A female client who has been sexually active for 5 years is found to have gonorrhea. The client is upset and asks the nurse, "What can I do to keep from getting another infection in the future?" Which statement by the client indicates that the teaching by the nurse was effective?

"My partner has to use a condom all the time."

A 17-year-old client tells the nurse that her sister had an ectopic pregnancy about 3 months ago and had to have her fallopian tube removed. The nurse determines that this young woman needs additional information when she states:

"My sister is lucky, because she won't have a period again."

A client asks the nurse what she should do if she forgets to take the pill one day. How should the nurse respond?

"On the next day take one pill in the morning and one before bedtime."

A genetic counselor is working with a couple, each of whom is a carrier of an autosomal recessive disorder. Which statement indicates that the couple has understood the teaching about this disorder?

"There is a 1-in-4 chance of having a child with the disorder." - Mendelian genetic theory, when both parents are carriers of an autosomal recessive disorder there is a 25% probability that a child will have the disorder. There is a 25% probability that a child born to this couple will have the disorder. The statement that none will have the disorder, indicates that the couple does not understand Mendel's theory of probability. When both partners are carriers there is a 50% probability that a child will be a carrier and a 25% probability that a child will have the disorder. If one of the parents has the disorder there is a 50% probability that a child will have the disorder.

While being admitted for a lumpectomy the client begins to cry and says, "I found the lump a few months ago, but I didn't go to the doctor because of what it could be." How should the nurse reply?

"This has been frightening for you."

A pregnant client has a positive group B Streptoccus (GBS) test at 36 weeks' gestation. What is the priority instruction that the nurse will include in the client's teaching plan?

"This information will be in your prenatal record, but please remind your labor and delivery nurse of this finding."

A client undergoes dilation and curettage (D&C) after an early miscarriage (spontaneous abortion). The nurse finds her crying later in the day. What is the most appropriate statement by the nurse?

"This must be a very hard experience for you to deal with."

After 5 years of unprotected intercourse, a childless couple comes to the fertility clinic. The husband tells the nurse that his parents have promised to make a down payment on a house for them if his wife gets pregnant this year. What is the nurse's best response to this comment?

"This must be very difficult for you with this added pressure."

An older female client tells the nurse in the clinic that she has a cystocele that was diagnosed a year ago. She has urinary frequency and burning on urination. The client asks, "The doctor wanted me to have surgery for the cystocele last year, but I can manage with peripads. It won't hurt not to have surgery, will it?" How should the nurse respond?

"Yes, you're risking kidney damage." -A cystocele is a herniation of the bladder through the vaginal wall resulting from weakened pelvic structures. In this condition the herniated bladder does not empty effectively and urinary stasis, chronic Infection; and renal failure may result. The surgery improves bladder function and prevents renal failure; it is needed. Bowel obstruction is a complication of a rectocele, not cystocele. Although corrective surgery will reduce perineal pressure, its primary purpose is to improve bladder function and prevent complications

A woman visits the clinic for an annual physical examination, and herpes genitalis is diagnosed. The client asks how the disease can be diagnosed without any tests. How should the nurse reply?

"You have blisters on the skin around your vagina." -Herpes genitalis is characterized by a cluster of vesicles, not one lesion or a rash or vaginal discharge. The characteristic sign of herpes genitalis is a cluster of vesicles (blisters) on the vulva, perineum, vagina, cervix, and/or perianal area. These rupture spontaneously, leaving painful erosions.

A multipara who is admitted to the hospital for repair of a rectocele and cystocele asks a nurse why these problems happened to her. How should the nurse respond?

"You have relaxation of the muscles in your lower pelvis."

A married couple has been using oral contraceptives to delay pregnancy. When the wife misses her regular menstrual period, she decides to find out whether she is pregnant. She tells the nurse that pregnancy may have occurred because she missed her contraceptive pills for 1 week when she had the flu. How should the nurse respond?

"You may be correct. The effect of contraceptive pills depends on their being taken on a regular schedule."

A young woman has been using oral contraceptives. When she misses her regular menstrual period, she visits the women's health clinic and tells the nurse that she may be pregnant because she missed taking her contraceptive pills for 1 week when she had the flu. How should the nurse respond?

"You may be right. One of the reasons that an exact schedule is prescribed for birth control pills is that they have to be taken regularly to be effective."

A woman visits the prenatal clinic because an over-the-counter pregnancy test has rendered a positive result. After the initial examination verifies the pregnancy, the nurse explains some of the metabolic changes that occur during the first trimester of pregnancy. (Select all that apply.)

1 Sleep needs increase. 2 Fluid retention increases. 4 Calcium requirements increase. -Estrogen increases the secretion of corticosteroids, which decrease the basal metabolic rate, resulting in fatigue. Sodium is retained, and fluid retention increases to meet total needs. During the first trimester approximately 1.2 g of calcium is needed each day; this need continues throughout pregnancy as the fetal skeleton is being formed. Body temperature increases because of the increased metabolism related to the growth of the fetus. Carbohydrate needs increase because the secretion of insulin by the pancreas is increased; however, insulin is destroyed rapidly by the placenta. The stress of pregnancy may precipitate gestational diabetes.

The nurse is teaching a sex education course to high school students. What should the nurse teach them about why gonorrhea is difficult to control? (Select all that apply.)

1 Symptoms of the disease are vague. 3 The incubation period is relatively short. 4 Causative organisms have become resistant to treatment. -Many clients with gonorrhea are asymptomatic. The incubation period is 3 to 5 days. There is no effective readily available blood test for gonorrhea. Gonorrhea responds well to treatment, but the Centers for Disease Control and Prevention has received several reports of resistant strains; at times backup secondary medications must be used. Urethral/vaginal smears or cultures are specific for the identification of the gonococcal organism.

The nurse discusses the recommended weight gain during pregnancy with a newly pregnant client who is 5 feet 3 inches tall and weighs 125 lb. The nurse explains that with the recommended weight gain, at term the client should weigh about:

1. 150 lb -A weight of 150 lb would put the client within the recommended weight gain of at least 25 lb for a woman who was of average weight for her height before pregnancy. A weight of 140 lb is less than the recommended weight gain for a woman of average weight for height before pregnancy, as are 135 lb and 130 lb.

A client who has just begun breastfeeding complains that her nipples feel very sore. What should the nurse encourage the mother to do? (Select all that apply.)

1. Apply cool packs to her breasts to reduce the discomfort 2. Take the analgesic medication prescribed to limit the discomfort 5 Assume a different position when breastfeeding to adjust the infant's sucking

A woman has made the decision to have breast augmentation surgery, and the procedure is to be performed on an outpatient basis. As part of the preoperative protocol, the nurse provides teaching regarding the discharge instructions. Which instructions apply to this type of surgery? (Select all that apply.)

1. Avoid taking aspirin or NSAIDs (e.g., ibuprofen [Advil]) for pain relief. 2 Sleep with your head and torso elevated for at least 1 week. 5 Take your temperature daily and notify the clinic if it goes above 99.6° F.

A nurse is teaching a class of expectant parents about changes that are to be expected during pregnancy. What changes does the nurse explain result from the melanocyte-stimulating hormone? (Select all that apply.)

1. Choasma 2. Linea nigra

In childbirth classes the nurse is teaching paced breathing techniques for use during labor. In which order should the breathing techniques be used as labor progresses?

1. Cleansing breaths 2. Slow, deep breaths 3. Modified paced breathing 4. Pant-blow breathing 5. Slow, exhalation pushing

Contraceptives that contain estrogen-like and/or progesterone-like compounds are prepared in a variety of forms. Which contraceptives should the nurse tell clients have a hormonal component? (Select all that apply.)

1. Oral drugs 6, Transdermal agents

A nurse caring for a pregnant client and her partner suspects domestic violence. Which observations support this suspicion? (Select all that apply.)

1. The woman has injuries to the breasts and abdomen. 3. The partner answers questions that are asked of the woman. 4. The woman has visited the clinic several times in the last month.

A client who had a child with Tay-Sachs disease is pregnant and is to have an amniocentesis to determine whether the fetus has the disease. The nurse counsels her to plan for the procedure at the optimal time for the procedure at:

14 to 16 weeks' gestation -An amniocentesis is done at this time because a therapeutic abortion may be legally and safely performed if desired by the parents. Six to 8 weeks' gestation is too early to perform an amniocentesis because the uterus has not ascended into the abdomen and there is little amniotic fluid present. Although an amniocentesis and therapeutic abortion may be performed at this time, it is preferred that they be done as early as possible. Twenty-two to 24 weeks' gestation is too late; the parents should not delay amniocentesis if they are considering a therapeutic abortion.

A client who menstruates regularly every 30 days asks a nurse on what day she is most likely to ovulate. Because the client's last menses started on January 1, the nurse should tell her that ovulation should occur on which day in January?

16

A nurse is discussing informed consent with a client who is scheduled for a hysterectomy. What should the informed consent include? (Select all that apply.)

2 Explanation of available alternative treatments 3 Answers to questions and concerns about the procedure 4 Complete description of the possible dangers and discomforts

A nurse is teaching a female client about using oral contraceptives and when to report complications. What clinical findings should alert the client to stop taking the contraceptive and notify the health care provider immediately? (Select all that apply.)

2 Headaches 4 Visual distrubances -Headaches, either sudden or persistent, may indicate hypertension or a cardiovascular event. Visual disorders, such as partial or complete loss of vision or double vision, may indicate neuro-ocular lesions, which are associated with the use of some oral contraceptives.

The nurse is obtaining a health history from a client with endometriosis. What consequences can occur as a result of this disorder? (Select all that apply.)

2 Metrorrhagia 4 Bowel strictures 5 Voiding difficulties -Metrorrhagia is a possible complication; bleeding between periods is due to the bleeding of endometrial tissue outside the uterus. The excessive tissue in endometriosis may impinge on the colon and cause ribbonlike stools. The endometrial tissue may impinge on the bladder and ureters and cause voiding difficulties. Menopause is a developmental adaptation. It does not complicate endometriosis, which is the presence of endometrial tissue outside the uterus that is prone to bleeding. Endometrial tissue does not enter the brain, so there are no cerebral complications.

A primigravida in her seventh week of gestation asks the nurse when she can expect to feel her baby move. The nurse replies that quickening usually occurs in the:

2. 20th week

Although a client in labor is prepared and plans to participate in the labor and birth process, she states that she is in severe discomfort. The nurse administers the prescribed butorphanol (Stadol). Which phase of labor is the safest time for the nurse to administer this medication?

2. Active phase -Although a client in labor is prepared and plans to participate in the labor and birth process, she states that she is in severe discomfort. The nurse administers the prescribed butorphanol (Stadol). Which phase of labor is the safest time for the nurse to administer this medication?

A nurse is planning a childbirth education class about maternal psychological and physiological changes as pregnancy nears term. Which problems and concerns should the nurse include in the presentation? (Select all that apply.)

2. Nesting needs increase. 4. Anxiety and childbirth increases. 5. Gastrointestinal motility decreases.

A client attending a prenatal class about nutrition tells the nurse that she is a strict vegetarian (vegan). What should the nurse encourage the client to eat that includes all the essential amino acids?

2. Whole-grain cereals and nuts

A 20-year-old woman is known to be heterozygous for the cystic fibrosis (CF) gene. Her husband's genotype is unknown at present and the couple is expecting their first child. What should the nurse tell the couple about the probability of their baby's having CF?

25% or less -Males with cystic fibrosis are usually sterile; therefore the father does not have cystic fibrosis, but he could be a carrier. If both parents are heterozygous carriers, the chance of having a child with CF is 25%. When one parent is a heterozygous carrier and the other has two unaffected genes, the chance of having a child who has CF is 0% but the chance of having a child who is a carrier is 50%. If both parents are heterozygous carriers or if one parent is a heterozygous carrier and one parent has two unaffected genes, the chance of having a child that is a carrier, not a child that is affected, is 50%.

The health care provider has prescribed 500 mg of cephalexin (Keflex) by mouth every 6 hours for 10 days for a client with mastitis. The health care provider has given the client 24 sample tablets of 250 mg apiece. How many days should this supply last? Record your answer using a whole number. _____ days

3

Which statements by a client with hyperemesis gravidarum would confirm that the client needs further teaching? (Select all that apply.)

3 "I'll start limiting my carbohydrates." 4 "I'll lie down for at least 2 hours after I eat."

An estrogen-progestin oral contraceptive is prescribed for a client. Which adverse effects should the nurse teach the client to report to the health care provider? (Select all that apply.)

3 Chest Pain 5 Breast soreness 6 Calf tenderness -Estrogen-progestin contraceptives have been associated with breast malignancy; clinical manifestations include breast soreness, thickening, and lack of symmetry. These contraceptives have also been associated with thrombophlebitis; clinical manifestations of thrombophlebitis include calf tenderness and redness and heat over the affected area. If the clot travels, it could present as a pulmonary embolism, so chest pain should be reported. Lethargy, dizziness, and constipation are not expected side effects of this medication

A client appears at the clinic after getting a positive result on a home pregnancy test. She states that her last menstrual period began 10 weeks ago. The client expresses fear because she has been recently found to have syphilis. What prescriptions will the nurse expect to receive from the health care provider because of this information? (Select all that apply.)

3 Screening and testing of sexual partners 5 Intramuscular benzathine penicillin G, 2.4 million units, one dose

A client who is taking an oral contraceptive calls the nurse with concerns about side effects of the medication. Which adverse effect of this medication should alert the nurse to inform the client to immediately stop the contraceptive and contact the health care provider? (Select all that apply.)

3. Visual disturbances 4. Persistent headaches

The nurse instructs a pregnant client in the sources of protein that can be used to meet the increased daily requirement during pregnancy. How many grams of protein should the client eat each day?

60 g

Which client is most at risk for osteoporosis?

A 66-year-old white woman, 5 foot 1 inch and 100 lb, who is a paralegal -A postmenopausal woman who is small-boned, thin, and relatively sedentary is at risk for osteoporosis; other risk factors are family history, and white or Asian ethnicity. The postmenopausal years are considered to be 65 years and older; however, each individual is unique. A perimenopausal woman who is relatively heavy and does not smoke is at less risk for osteoporosis than is a thin postmenopausal woman. The perimenopausal years are considered to be 45 to 64 years of age; however, each individual is unique. Postmenopausal women who are black are at lower risk for osteoporosis than are white and Asian women. A perimenopausal woman who takes a daily calcium supplement is at less risk for osteoporosis than a woman who does not take a calcium supplement.

A couple at the prenatal clinic for a first visit tells the nurse that their 2-year-old child has just been found to have cystic fibrosis. They state there is no family history of this disorder. They ask the nurse about the chances of their having another child with cystic fibrosis. Knowing that this disorder has an autosomal recessive mode of inheritance, how should the nurse respond?

There is a 25% chance the baby will be affected and a 50% chance that the baby will be a carrier.

A 16-year-old girl who has become sexually active asks the nurse, "What's the most effective way to prevent a pregnancy?" Which method of preventing pregnancy should the nurse tell her is most effective?

Abstaining from sexual intercourse

After a client gives birth she has the following vital signs: temperature 99.4° F (37.4° C); pulse rate 80 beats/min and regular; respiratory rate of 16 breaths/min, with even respirations; and blood pressure of 148/92 mm Hg. Which vital sign should the nurse continue to monitor?

Blood pressure

After the client gives birth her vital signs are temperature 99.3° F (37.4° C); pulse 80 beats/min, regular and strong; respirations 16/min, slow and even; and blood pressure 148/92 mm Hg. Which vital sign should the nurse check more frequently?

Blood pressure

A woman who was discharged recently from the hospital after undergoing a hysterectomy calls the clinic and states that she has tenderness, redness, and swelling in her right calf. What should the nurse instruct the client to do?

Call an ambulance to go to the emergency department.

Which behavior indicates to a nurse that a new mother is in the taking-hold phase?

Calling the baby by name -The mother has moved into the taking-hold phase when she takes control and becomes actively involved with her infant and calls the infant by name. She has completed the taking-in phase when her own needs no longer predominate. Talking about the labor and birth in the taking-in phase when she has the need to integrate the experience. Touching the baby with her fingertips is the initial early action of the taking-in phase. Being involved with the infants need to eat and sleep is part of the taking-in phase.

A nurse is caring for a client with tertiary syphilis. Which body system should the nurse monitor most closely?

Cardiovascular

At 6 weeks' gestation a client is found to have gonorrhea. What medication does a nurse expect the health care provider to prescribe?

Ceftriaxone (Rocephin)

A practitioner orders doxycycline (Vibramycin) for a sexually active woman with a history of a mucopurulent discharge and bleeding associated with cervical dysplasia, dysuria, and dyspareunia. With which sexually transmitted infection are these clinical findings and medication therapy commonly associated?

Chlamydial infection -The signs and symptoms listed and the treatment ordered (doxycycline or azithromycin) indicate that the client has a chlamydial infection. Painful blisters on the genitalia, fever, malaise, dysuria, and dyspareunia are signs of herpes simplex virus 2 infection. Chancre formation is a sign of primary syphilis; a symmetrical rash accompanied by malaise, fever, anorexia, and headache is indicative of secondary syphilis. Dysuria, heavy greenish-yellow purulent discharge, and swollen Bartholin glands are signs of gonorrhea.

A client making her first visit to the prenatal clinic asks which immunization can be administered safely to a pregnant woman. What should the nurse tell her?

Inactive influenza -The inactive influenza and diphtheria, tetanus, pertussis (dTAP) can be safely administered during the first trimester of pregnancy, although dTAP is recommended at 27 to 36 weeks' gestation to provide immunity to the mother and infant. Rubella (measles) and rubeola (German measles) are both live viruses that should never be administered during pregnancy because they can have teratogenic effects. The inactivated influenza vaccine may be given because it is a killed virus vaccine and will not have a teratogenic effect.

A client who suspects that she is 6 weeks pregnant appears mildly anxious as she is waiting for her first obstetric appointment. What symptom of mild anxiety does the nurse expect this client to experience?

Increased alertness -Increased alertness is an expected common behavior that occurs in new or different situations when a person is mildly anxious. The other options are a common sign of moderate to severe anxiety.

A client who wishes to postpone having children until she and her husband are financially sound tells the nurse she has been taking oral contraceptive pills for several years. What finding indicates a potential risk in regard to continued use of birth control pills?

Increased blood pressure

A hysterectomy is scheduled for a client with endometrial cancer. Before the surgery, what should the nurse prepare the client to expect?

Indwelling urinary catheter

A nurse caring for a pregnant woman determines that she is engaging in the practice of pica. Why should the nurse prepare a teaching plan for this client?

Inedible items are being ingested. -Pica is the eating of inedibles such as starch or dirt. There is a cultural influence on this practice, but it may also be related to malnutrition or anemia. Food cravings frequently occur in pregnant women. If many foods are causing nausea and vomiting, the client has morning sickness. If it continues past the first trimester, it may be hyperemesis gravidarum. The dislike for essential food groups does not describe the practice of pica.

A client who is visiting the prenatal clinic for the first time has a serology test for toxoplasmosis. What information about the client's activities in the history indicates to the nurse that there is a need for this test?

The client takes care of a cat.

A female client with Hodgkin disease is to start total nodal irradiation. She and her partner, who are planning a family, become concerned when they learn that the radiation therapy includes the pelvic area. Before responding what must the nurse consider?

The eggs in the ovaries can be removed and frozen for future use.

A client underwent a mastectomy 24 hours ago. Which information will the nurse include in the plan of care?

The hand and elbow of the affected arm will be elevated above the shoulder.

After a client undergoes a biopsy for suspected cervical cancer, the laboratory report reveals a stage 0 lesion. What does a nurse conclude about this client's stage of cancer?

The lesion is carcinoma in situ.

A client who has just had a cesarean birth is receiving IV fluids and has an indwelling catheter. The client's fluid intake will need to be increased when the nurse identifies:

Dark amber urine -A dark amber or tea color indicates highly concentrated urine and requires additional hydration of the client.

A 24-year-old client complains to the nurse in the women's health clinic that her breasts become tender before her menstrual period. What should the nurse recommend that the client do 1 week before an expected menses?

Decrease caffeine intake -The client is exhibiting one symptom of premenstrual syndrome (PMS); eliminating food and beverages containing caffeine can limit breast swelling. Salt intake should be reduced premenstrually to limit the development of edema. Increased protein intake is unnecessary if the client is eating a nutritious diet. Exercise should be increased before the menstrual period to help ease the symptoms of PMS.

A client's temperature is 100.4° F 12 hours after a spontaneous vaginal birth. What does the nurse suspect is the cause of the increased temperature?

Dehydration

The nurse explains to a woman in her 24th week of pregnancy that absorption of medications taken orally during pregnancy may be altered as a result of:

Delayed gastrointestinal emptying

A client who recently was told by her practitioner that she has extensive terminal metastatic carcinoma of the breast tells the nurse that she believes an error has been made. She states that she does not have breast cancer, and she is not going to die. The nurse determines that the client is experiencing the stage of death and dying known as:

Denial

A 20-year-old developmentally disabled woman is a resident in a group home. She has had four abortions in the past 2 years, and the agency supervisor recommends that she be sterilized. It is obvious that the client is unable to exercise informed consent for sterilization. The nurse understands that the procedure cannot be performed without legal consent from the:

Court-appointed individual or group

The partner of a woman experiencing back pain in labor asks what he can do to help. The nurse demonstrates how to apply counterpressure to his partner's back. Where on the image should counterpressure be applied?

D Counterpressure on the lower sacrum relieves discomfort by lifting the fetal head off the spinal nerves when the fetus is in a posterior position.

At a client's first prenatal visit, the healthcare provider performs a pelvic examination, stating that the client's cervix is bluish purple, which is known as the Chadwick sign. The client becomes concerned and asks whether something is wrong. The best response is "This is expected; it:

Is caused by increased blood flow to the uterus during pregnancy" -Stating that the Chadwick sign is caused by increased blood flow to the uterus during pregnancy underscores the normalcy of Chadwick's sign and provides a simple explanation of the cause; women often need reassurance that the physical changes associated with pregnancy are expected. Stating that the Chadwick sign helps confirm pregnancy answers part of the question but fails to explain why it occurs. The Chadwick sign is a probable sign of pregnancy; it is not seen in nonpregnant women. There is no free blood circulating in the uterus during pregnancy.

A nurse has learned that infants born to very young mothers are at risk for neglect or abuse primarily because an adolescent characteristically:

Is involved in seeking her own identity -Adolescent parents are still involved in the developmental stage of resolving their own self-identity; they have not sequentially matured to intimacy and generativity, making nurturing of another difficult.

A pregnant client with severe preeclampsia is receiving an infusion of magnesium sulfate. What does the nurse identify as the main reason that this medication is administered?

It acts as an anticonvulsant.

When a mother sees her newborn son assume a fencing position as she turns his head, she becomes concerned. What should the nurse explain about this reflex?

It is expected in the healthy newborn.

A client starting her second trimester asks a nurse in the prenatal clinic whether she can safely take an over-the-counter (OTC) medicine now that she is past the first 3 months of pregnancy. The nurse explains why she should consult with her health care provider before taking any oral medications. What physiological alteration associated with pregnancy may change the client's response to medication?

Longer gastrointestinal emptying time -Gastrointestinal motility is reduced during pregnancy because of the high level of placental progesterone and displacement of the stomach superiorly and the intestines laterally and posteriorly; absorption of some drugs, vitamins, and minerals may be increased because of their slow passage through the gastrointestinal tract. The glomerular filtration rate increases during pregnancy. The amount of gastric secretion is somewhat lower in the first and second trimesters; it increases in the third trimester. The development of fetal-placental circulation is unrelated to the absorption of drugs.

A 28-year-old woman is scheduled to undergo a laparoscopic bilateral salpingo-oophorectomy. What does a nurse expect to be the client's priority concern?

Loss of childbearing potential

A 30-year-old woman is to undergo total abdominal hysterectomy for noninvasive endometrial cancer. The nurse anticipates the client may have difficulty adjusting emotionally to this type of surgery. What is the most common reason for this difficulty?

Loss of femininity

A nurse is caring for a client with vaginal bleeding caused by placenta previa. What is the best nursing intervention to delay the birth of the fetus?

Maintaining bed rest

During discharge teaching a client who just had a hysterectomy states, "After this surgery, I don't expect to be interested in sex anymore." What should the nurse consider before responding?

Many women incorrectly equate hysterectomy with loss of libido. -The uterus is often erroneously believed necessary for a satisfying sex life. Sexuality after hysterectomy should not be diminished, particularly because the fear of pregnancy no longer exists. Although the estrogen level is reduced, libido is influenced by psychological as well as hormonal factors. Although body image changes can interfere with sexuality, this is not an expectation for most women.

Thirty minutes after a client gives birth, the nurse palpates the client's uterus. It is relaxed and the lochia is excessive. What is the nurse's initial action?Massage the uterus

Massage the uterus

One hour postpartum a nurse evaluates the amount of vaginal bleeding and determines that a client's uterus has become relaxed and boggy. What should the nurse do next?

Massage the uterus until firm

A nurse is caring for a client who has contracted a trichomonal infection. Which oral drug should the nurse anticipate that the health care provider will most likely prescribe?

Metronidazole (Flagyl) -Metronidazole (Flagyl) is a potent amebicide. It is effective in eradicating the protozoan Trichomonas vaginalis. Penicillin is administered for its effect on bacterial, not protozoal, infections. Gentian violet is a local anti-infective that is applied topically; it may cause discoloration of the skin. It is effective against Candida albicans. Nystatin (Mycostatin) is an antifungal for infections caused by C. albicans.

A nurse understands the stages of parental adjustment that follow the birth of an at-risk infant who is in the neonatal intensive care unit. To better plan nursing care, the nurse bases observations and assessments on the recognition that the:

Mother should be reunited with her infant as soon as possible to enhance adjustment

What must the LPN observe first when planning to promote mother-infant attachment?

Mother-infant interaction

The nurse teaches a pregnant client why she needs a folic acid supplement. Which neonatal disorder does folic acid prevent?

Neural tube defects -A folic acid supplement (0.4 mg/day) greatly reduces the incidence of fetal neural tube defects.

A female client is scheduled for a hysterectomy. While discussing the preoperative preparations, the nurse determines that the client's understanding of the surgery is inadequate. What is the next nursing intervention?

Notifying the surgeon that the client needs more information -Legally the person performing the surgery is responsible for informing the client adequately; the nurse may clarify information, witness the client's signature, and co-sign the consent form. Describing the proposed surgery to the client is beyond the scope of nursing practice. The nurse could face criminal charges of assault and battery for proceeding when there is a lack of informed consent . Explaining gently that she should have asked more questions places blame on the client; it is the responsibility of the surgeon to impart the vital information required for consent.

A client who has had a cesarean birth seems upset. She has been having difficulty breastfeeding for 2 days and now asks the nurse to bring her a bottle of formula. What is the nurse's initial action?

Observing the client's breastfeeding technique -The nurse should assess the client to determine why she is having difficulty with breastfeeding. She may be uncomfortable or in need of assistance with her breastfeeding technique. Immediately providing the formula without assessing the situation does not meet the client's needs at this time.

A client who has breast cancer had postlumpectomy chemotherapy and is now scheduled for radiation on an outpatient basis. What is an important nursing intervention while the client is receiving radiation?

Observing the irradiated site daily for redness or irritation

A primigravida in the first trimester tells a nurse that she has heard that hormones play an important role in pregnancy. Which hormone should the nurse tell the client maintains pregnancy?

Progesterone

Which factor in a client's history increases the risk for osteoporosis?

Prolonged immobility

A client in her 30th week of gestation is in preterm labor, and the practitioner prescribes betamethasone (Celestone). The client asks the nurse why she is being given this drug. As a basis for the response the nurse takes into consideration that it:

Promotes neonatal pulmonary maturity -Betamethasone (Celestone), a corticosteroid, accelerates lung maturity and reduces intravascular hemorrhage and necrotizing enterocolitis in the preterm neonate if given 24 hours before birth.

A client who is in preterm labor at 34 weeks' gestation is receiving intravenous tocolytic therapy. The frequency of her contractions increases to every 10 minutes, and her cervix dilates to 4 cm. The infusion is discontinued. Toward what outcome should the priority nursing care be directed at this time?

Promotion of maternal and fetal well-being during labor

What antidote to the side effects of terbutaline (Brethine) should a nurse have available?

Propranolol (Inderal) -Propranolol (Inderal) is a beta-blocking agent that reverses the uterine inhibitory responses and cardiovascular effects of terbutaline (Brethine). Levodopa (L-dopa) is not an antidote for terbutaline; it is used in Parkinson's disease. Furosemide (Lasix) is a diuretic; it will not reverse the cardiovascular effects indicated. Ritodrine (Yutopar) may cause responses similar to those of terbutaline; it is sometimes used to halt premature labor because it inhibits beta2- receptors

A nurse is obtaining a health history from a client with newly diagnosed cervical cancer. What aspect of the client's life is most important for the nurse to explore at this time?

Support system

A husband who is coaching his wife during labor demonstrates an understanding of the transition phase of labor when, as his wife starts to push with each contraction, he instructs her to:

Take quick, shallow breaths, and then blow

An adolescent who gave birth one day ago confides to the nurse that she hopes that her baby will be good and sleep through the night. What should the nurse include in the plan of care to facilitate a realistic expectation of a nighttime newborn schedule?

Talk softly and cuddle the baby when crying occurs

The nurse reads the history of a neonate admitted to the nursery and discovers that the infant's mother was listed as gravida 1 para 1 before the baby was born. How should the nurse use these data to gather more information?

To consider that someone recorded the gravida and para incorrectly -Gravida refers to pregnancies, including this one, and para refers to pregnancies terminated (by whatever means) after the age of viability. If this is the client's only pregnancy (gravida 1) she could not have had a previous pregnancy that ended after the age of fetal viability. Para will not exceed gravida. One pregnancy is gravida 1. A twin pregnancy is still one pregnancy terminated after the age of viability. Because the documentation of the client indicates that she is gravida 1, it cannot be assumed that it is the woman's second pregnancy.

A client at 16 weeks' gestation is scheduled for a sonogram followed by amniocentesis. The nurse instructs the client to drink 8 oz of fluid and not void before the sonogram. The nurse explains that this is done:

To improve visualization of the fetus

A nurse is teaching a client how to self-administer a medicated douche. In which direction should the nurse instruct the client to direct the douche nozzle?

Toward the sacrum -Toward the sacrum is the anatomic direction of the vaginal tract in the back-lying position. The vaginal tract may be injured if the douche nozzle is directed without consideration of the pelvic anatomy.

A client with osteoporosis has been receiving dietary information from the nurse. Which food selection by the client indicates that the nurse's dietary instruction was effective?

Turnip greens -Turnip greens are high in calcium and vitamins. A high level of nitrogen from protein breakdown may increase the release of calcium from bone to serve as a buffer of the nitrogen

A nurse instructs a client who is taking oral contraceptives to increase her intake of dietary supplements. Which supplement should be increased?

Vitamin C -Oral contraceptives can affect the metabolism of certain vitamins, particularly vitamin C, and supplementation may be required.

A thin 24-year-old woman who runs 10 miles each week asks the nurse for advice about preventing osteoporosis. Which vitamin and other dietary supplement should the nurse recommend?

Vitamin D and calcium citrate

A client who has had recurrent infections before and during pregnancy should be instructed to eat a nutrient-rich diet as a means of supporting the body's natural defense mechanisms. What should the nurse encourage the client to include in her diet?

Vitamins A, C, and E and selenium -Vitamins A, C, and E and selenium are immune-stimulating nutrients.

A nurse is teaching a breastfeeding mother about cleansing her nipples. What technique should the nurse emphasize?

Wash the breasts and nipples with water when bathing.

A nurse is teaching a class about childbearing and contraceptive options. The nurse explains that fertilization of the ovum by the sperm occurs at a specific time. When does it occur?

When one sperm penetrates the wall of the ovum

What client behavior indicates to the nurse that a woman needs further teaching about breastfeeding her newborn?

When she leans forward to place her breast in the infant's mouth

A client is to undergo a tuberculin test as part of her prenatal workup. Before administering the test, what information about the client should the nurse obtain?

Whether an earlier tuberculin test's result was positive

What is most important information to teach to a client who has had a mastectomy before she leaves the hospital?

Why self-examination of the remaining breast is important

What should the nurse emphasize in a class about childbirth?

Education; exercise, and breathing techniques

The nurse should explain to the newly pregnant primigravida that the fetal heartbeat will first be heard with:

Electronic Doppler ultrasonography at 10 to 12 weeks

A client who underwent mastectomy because of breast cancer is now undergoing chemotherapy, which has caused hair loss. The client states, "I feel like I've lost my sense of power." What is the nurse's best response?

"Losing power seems important to you."

A 35-year-old client is scheduled for a vaginal hysterectomy. She asks the nurse about the changes she should expect after surgery. How should the nurse respond?

"A hysterectomy doesn't affect the chronological age when menopause usually occurs." -As the term hysterectomy implies, only the uterus is removed. The ovaries remain; therefore, the client will experience menopause around the same time as women who have functioning ovaries. The client will ovulate and will not experience surgical menopause because the ovaries are not removed with a hysterectomy. There should be no discomfort if there is an appropriate period of healing before the resumption of sexual intercourse.

While teaching a prenatal class about infant feeding, the nurse is asked about the relationship between breast size and ease of breastfeeding. How should the nurse respond?

"Breast size is not related to milk production."

A postpartum client is scheduled to have a tubal ligation. She has asked that her husband not be told about the procedure because she has told him that she is having exploratory surgery. The client's husband asks the nurse why his wife needs to have exploratory surgery. How should the nurse respond?

"Have you talked to your wife about your concerns?"

What statement helps the nurse determine that a woman with genital herpes (HSV-2) understands her self-care in regard to this infection?

"I must be careful when I have sex because herpes is a lifelong problem." -Genital herpes (HSV-2) is characterized by remissions and exacerbations; it cannot be cured. Most pregnant women with HSV-2 have cesarean deliveries to prevent their newborns from contracting the disease while passing through the vagina. Clients should abstain from sex until 10 days after the lesions heal. Herpes can be controlled, not cured.

A pregnant client whose first child has Down syndrome is about to undergo amniocentesis. The client tells the nurse that she does not know what she will do if this fetus has the same diagnosis. The client asks the nurse, "Do you think abortion is the same as killing?" How should the nurse respond?

"I really can't answer that question. Are you ambivalent about abortion?"

After 8 postpartum hours the nurse determines that a client's fundus is 3 cm above the umbilicus and displaced to the right. Which statement is most significant in confirming the reason for the location of the uterus?

"I went to the bathroom, but I can't seem to urinate." -Not being able to urinate, in conjunction with the other findings, suggests urine retention. Thirst is unrelated to the other findings; it is related to dehydration. Changing the pad once in 8 hours is an expected postpartum response. Contractions while breastfeeding are expected; oxytocin secretion, which is stimulated by infant suckling, causes the uterus to contract.

A premenopausal client is scheduled for a total abdominal hysterectomy. Which statement indicates to the nurse that the preoperative teaching has been understood?

"I won't have any more menstrual periods after this surgery." - Correct4 "I won't have any more menstrual periods after this surgery." Menstrual flow is the shedding of the endometrial lining of the uterus, and a hysterectomy is the removal of the uterus

The nurse determines that a young female client who is being treated for a sexually transmitted infection (STI) understands instructions regarding future sexual contacts. Which statement confirms the nurse's conclusion?

"I won't have unprotected sex again, and I'll tell my partners to be tested for STIs."

A nurse from the pediatric clinic who is strongly opposed to any chemical or mechanical method of birth control is asked to work in the family planning clinic. What is the most professional response that this nurse could give to the requesting supervisor?

"I would prefer another assignment that is not contrary to my beliefs."

After a client has a spontaneous abortion at 12 weeks' gestation, the nurse notes that she and her partner are visibly upset. The partner has tears in his eyes, and the client is sobbing quietly with her face turned to the wall. At this time, what is the nurse's most therapeutic statement?

"I'll be here if you want to talk."

A nurse is teaching a postpartum client the characteristics of lochia and any deviations that should be reported immediately. What client statement indicates that the teaching was effective?

"I'll notify the clinic if my lochia starts to smell bad." -Lochia has a characteristic menstrual musky or fleshy smell. A foul-smelling discharge, along with fever and uterine tenderness, suggests an infection. Passing clots is a common occurrence. Lochia changing from red to pink is expected as lochia rubra progresses to lochia serosa. Although many women have a minimal discharge after 2 weeks, it is not uncommon for lochia alba to last 6 weeks.

After 18 months of unsuccessful attempts at conception by a client, primary infertility related to anovulatory cycles is diagnosed. Clomiphene citrate (Clomid) is prescribed. The nurse concludes that the client understands the teaching about the correct time to take the clomiphene when the she states:

"I'll start the pills on the fifth day of my cycle." -The objective is to stimulate ovulation near the 14th day of the menstrual cycle, and this is achieved by taking the medication on the fifth through the ninth days; there is an increase in two pituitary gonadotropins luteinizing hormone and follicle-stimulating hormone, with subsequent ovarian stimulation.

A client seeking advice about contraception asks a nurse about how an intrauterine device (IUD) prevents pregnancy. How should the nurse respond?

"It produces a spermicidal intrauterine environment." -Intrauterine devices produce a spermicidal intrauterine environment. A copper IUD (ParaGard T380A) inflames the endometrium, damaging or killing sperm and preventing fertilization and/or implantation; a Mirena IUD (LNG-IUS) releases levonorgestrel, damaging sperm and causing the endometrium to atrophy, thus preventing fertilization and/implantation. A diaphragm blocks the cervical os.

A primigravida is admitted to the birthing suite at term with contractions occurring every 5 to 8 minutes and a bloody show. She and her partner attended childbirth preparation classes. Vaginal examination reveals that the cervix is dilated 3 cm and 75% effaced. The fetus is at +1 station in occiput anterior position, and the membranes are intact. The client is cheerful and relaxed and asks the nurse whether it is all right for her to walk around. In light of the nurse's observations regarding the contractions and the client's knowledge of the physiology and mechanism of labor, how should the nurse respond?

"It's all right for you to walk as long as you feel comfortable and your membranes are intact."

What instruction should a nurse include when teaching about the correct use of a female condom?

"Remove the condom before standing up."

A client has just given birth to an infant with Down syndrome. The mother is crying and asks the nurse what she is supposed to do now. What is the nurse's best response?

"Tell me what you know about Down syndrome."

Which statement made by a pregnant client to a nurse indicates that the client does not understand the teaching about fetal growth and development?

"The baby gets food from the amniotic fluid." -The amniotic fluid serves as a protective environment; the fetus depends on the placenta, along with the umbilical blood vessels, for nutrients and oxygen. "The baby is smaller if the mother smokes," "The baby's oxygen is provided by the mother," and "The baby's umbilical cord has two arteries and one vein" are all true statements, and further teaching would not be required.

During a childbirth class, several participants have questions about the elective induction of labor. One participant states that it is more convenient for a woman with a busy schedule. What evidenced-based information should the nurse provide to the participant?

"The widespread use of elective induction increases the risk of unfavorable outcomes."

A client at 37 weeks' gestation gives birth to a healthy boy. While inspecting her newborn in the birthing room, the client becomes concerned and asks, "What's this sticky white stuff all over him?" How should the nurse respond?

"This is vernix. It helps protect the baby while he's in the uterus." -A factual response will allay the mother's concern. Vernix caseosa is a cheesy white substance that covers the fetus and confers protection from the amniotic fluid while the fetus is in utero. Most of it disappears by 40 weeks' gestation. Milia are white pinpoint dots (sebaceous glands) on the newborn's nose, chin, and forehead that disappear within a few weeks. The nurse should explain only what vernix is; referring to the infant as preterm may unnecessarily alarm the mother.

A 37-year-old woman is admitted to the unit with severe menorrhagia. During a conversation, the nurse learns that the client has a history of fibroids, menorrhagia, pelvic pain, and depression. The client has been undergoing hormone therapy in hopes of easing the symptoms and reducing fibroids' size, without success. The lab reports hemoglobin and hematocrit readings of 6.8 and 20.2, respectively. The client begins to sob and cries, "I don't know what to do — the doctor is recommending a hysterectomy, but I haven't had children yet!" What is the best response by the nurse?

"This must be so difficult for you. Children are really important to you?"

As the nurse helps a postpartum client change her perineal pad, the client comments, "I wish you didn't have to look at the pad it's embarrassing for me." What is the best nursing response?

"This seems to be uncomfortable for you, but I have to estimate the amount of blood loss to prevent any problems."

A nurse is giving discharge instructions to a new mother. What is the most important instruction to help prevent postpartum infection?

"Wash your hands before and after changing your sanitary napkins."

A client who is pregnant for the first time and is carrying twins is scheduled for a cesarean birth. What should the nurse tell the client to expect?

"We'll be encouraging you to walk early after surgery."

When encouraging a client to cough and deep breathe after a bilateral mastectomy, the client says, "Leave me alone! Don't you know I'm in pain?" What is the nurse's most therapeutic response?

"What's your pain level right now?"

A client visiting the prenatal clinic for the first time tells the nurse that she has heard conflicting stories about sex during pregnancy and asks about continuing sexual activity. How should the nurse respond?

"With an uncomplicated pregnancy, there are no limitations on sexual activity."

A postpartum client tells the nurse that she wishes to breastfeed. When the nurse brings her newborn to be breastfed, the client asks whether she may drink a small glass of wine to help her relax. How should the nurse respond?

"You seem a little tense. Tell me how you feel about breastfeeding." -Stating that the client seems tense and initiating a discussion honors the client's feelings and encourages expression of them; there is no reference to alcohol consumption and its relaxing effects. Alcohol ingestion should not be encouraged because it enters the breast milk.

When a client who had a mastectomy sees her incision for the first time, she exclaims, "I look horrible! Will it ever look better?" What is the nurse's best response?

"You seem shocked by the way you look now."

A client asks the nurse at the family planning clinic whether contraception is needed while she is breastfeeding. How should the nurse reply?

"You should use contraceptives, because ovulation may occur without a period." -Anovulation occurs in nursing mothers for varying periods; breastfeeding is not a reliable method of birth control. Periods may not occur for several months; sexual relations need not be delayed this long. Ovulation can occur without menstruation. Lactation may delay menses but does not reliably suppress ovulation.

During the postpartum period a client with heart disease and type 2 diabetes asks a nurse, "Which contraceptives will I be able to use to prevent pregnancy in the near future?" How should the nurse respond?

"You should use foam with a condom to prevent pregnancy — this is the safest method for women with your illnesses."

A client who is scheduled to have a hysterectomy starts to sob and says, "I told my husband that after this operation, I'll be only half a woman. He told me not to worry, but I know that he was just putting up a front." How should the nurse respond?

"You think this operation will affect how your husband feels about you as his wife."

A new mother who wishes to breastfeed her infant asks a nurse whether she needs to alter her diet. How should the nurse respond?

"You'll need extra amounts of the same foods you've been eating, plus more fluids."

A new mother wishes to breastfeed her infant and asks the nurse whether she needs to alter her diet. How should the nurse respond?

"You'll need greater amounts of the same foods you've been eating and more fluids."

A 63-year-old woman with the diagnosis of estrogen-receptor positive cancer of the breast undergoes lumpectomy and radiation therapy, and tamoxifen (Nolvadex) is prescribed. The client asks the nurse how long she will have to take the medication. The nurse responds:

"You'll need to take it for 5 years, after which it will be discontinued." -Tamoxifen is an estrogen antagonist antineoplastic medication that has been found to be effective in 50% to 60% of women with estrogen receptor-positive cancer of the breast. After 5 years of administration there is an increased risk of complications and the drug is discontinued. Tamoxifen usually is prescribed for 5 years after initiation of therapy, not for the rest of the client's life.

The clinic nurse is reviewing the dietary intake of a 16-year-old who is 12 weeks pregnant. What is the nurse's next action?

-Asking the client, "How many servings of dairy do you generally consume each day?" -Once the nurse knows the number of servings of calcium-rich foods the client consumes each day, the nutrition teaching plan can be personalized to her needs. The daily required intake of calcium for clients under 19 is 1300 mg. This amount can be obtained from 4 cups of milk or yogurt per day. Because there are sources of calcium other than milk, asking whether the client likes milk is not an appropriate first question. *Alternative sources of calcium include cheese, yogurt, figs, kale, sardines, orange juice with added calcium, creamy pesto, and cheese sauce.*

A nurse is to administer 5000 units of heparin to a prenatal client on prolonged bedrest. The label indicates that there are 20,000 units of heparin in each milliliter of solution. How much solution should the nurse give the client? Include a leading zero if applicable. Record your answer using two decimal places. ___ mL

0.25

Four days after a vaginal hysterectomy a client calls the follow-up service and tells the nurse that she has a yellowish-green vaginal discharge. The nurse advises the client to return to the clinic for an evaluation. Which symptoms are suggestive of a vaginal infection? (Select all that apply.)

1 Abdominal pain 3 Rising temperature

A nurse is teaching a prenatal class about the changes that occur during the second trimester of pregnancy. What cardiovascular changes should the nurse include? (Select all that apply.)

1 Cardiac output increases. 2 Blood pressure decreases. 3 The heart is displaced upward.

The nurse teaches a client about the increased need for vitamin A to meet the demands imposed by rapid fetal tissue growth during pregnancy. Which foods should the nurse encourage the client to ingest to meet this increased need? (Select all that apply.)

1 Carrots 4 Sweet potatoes

Which risk factors are associated with the future development of osteoporosis in women? (Select all that apply.)

1 Cigarette smoking 4 Familial predisposition 5 Inadequate intake of dietary calcium

A client undergoes anterior and posterior surgical repair of a cystocele and rectocele and returns from the postanesthesia care unit with an indwelling catheter in place. What should the nurse tell the client about the primary reasons for the catheter? (Select all that apply.)

1 Discomfort is minimized. 3 Retention of urine is prevented. 4 Pressure on the suture line is relieved.

A client asks the nurse about the use of an intrauterine device (IUD) for contraception. What information should the nurse include in the response? (Select all that apply.)

1 Expulsion of the device 2 Occasional dyspareunia 4 Risk for perforation of the uterus

A client at 7 weeks' gestation tells the nurse in the prenatal clinic that she has been bothered by episodes of nausea, but no vomiting, throughout the day. What should the nurse recommend? (Select all that apply.)

1 Focus on and repeat a rhythmic chant. 5 Eat small, frequent meals and eat dry crackers in between.

A nurse is assessing a client who is being admitted for surgical repair of a rectocele. What signs or symptoms does the nurse expect the client to report? (Select all that apply.)

1 Painful intercourse 3 Bearing-down sensations -The posterior vaginal wall is pushed forward by the herniation of the rectum; this protrusion causes painful intercourse. The posterior vaginal wall is pushed forward by the herniation of the rectum; this protrusion increases rectal pressure and causes the bearing-down sensation. A rectocele is not accompanied by abdominal pain. Urinary stress incontinence is the primary sign of a cystocele. A cystocele, not a rectocele, is associated with urinary tract infections.

A nurse is caring for an adolescent in labor an hour after she was admitted to the birthing unit. The adolescent is anxious and tense. She cries during contractions and asks the nurse for epidural anesthesia. The nurse obtains the adolescent's current vital signs and reviews her history and admission information. What nursing interventions are essential before epidural anesthesia is administered? (Select all that apply.)

1 Performing a baseline vaginal examination 2 Telling the adolescent what to expect with each procedure 3 Identifying risk factors that contraindicate epidural anesthesia

A nurse is caring for a client who is receiving internal radiation for cancer of the cervix. For which adverse reactions to the radiotherapy should the client be monitored? (Select all that apply.)

2 Hemorrhage 5 Increased temperature -Excessive sloughing of tissue may cause hemorrhage and is considered an adverse reaction. Infection, marked by an increase in temperature, may also develop from excessive sloughing of tissue. Nausea is an expected side effect of internal radiotherapy.

A 2-day-old infant who weighs 6 lb (2722 g) is fed formula every 4 hours. Newborns need about 73 mL of fluid per pound of body weight each day. In light of this information, approximately how much formula should the infant receive at each feeding?

2 to 3 oz -Infants require about 73 mL of fluid per pound and 60 calories a day per pound for growth. The infant's weight of 6 lb × 73 mL of fluid = 438 mL. If fed every 4 hours the infant will have six feedings: 438 ÷ 6 = 73 mL; 73 ÷ 30 (30 mL/oz) = 2.4 oz. Therefore the infant should be offered 2 to 3 oz per feeding. One or 2 oz is inadequate for this newborn. Three to 5 oz is excessive for this newborn.

A client is found to have pelvic inflammatory disease, and the health care provider prescribes intravenous cefotetan 2 g twice a day. The instructions on the vial of cefotetan say to reconstitute 20 mL of diluent to yield 1 g/10.5 mL. How much solution should the nurse add to the 100-mL bag of 0.9% sodium chloride? Record your answer using a whole number. ___ mL

21 mL

A 30-year-old client with a 35-day menstrual cycle is trying to become pregnant. The nurse counsels the client and her partner about the optimal timing of intercourse during the cycle. The nurse determines that the counseling has been effective when the couple state that they should have intercourse on the:

21st day of the cycle -Ovulation usually occurs 14 days before menses; in a 35-day cycle, ovulation may occur as late as the 21st day. Day 12 day of the cycle is the proliferative phase of the cycle; ovulation has not yet occurred. If the woman had a 28-day cycle, ovulation is expected 14th day of the cycle. By the 25th day of the cycle, the ovum in this woman has passed out of the fallopian tube and can no longer be fertilized.

The school nurse is teaching a group of 16-year-old girls about the female reproductive system. One student asks how long after ovulation it is possible for conception to occur. The most accurate response by the nurse is based on the knowledge that an ovum is no longer viable after:

24 hours

What should a nurse include in the teaching plan for a couple seeking information about family planning?

A condom must be held in place by the rim when the penis is withdrawn from the vagina.

A client is scheduled for a vacuum aspiration abortion to terminate an unwanted pregnancy. What information should the nurse's teaching plan include?

A temperature of 100.4° F (38° C) or higher should be reported immediately.

A husband sits in the waiting room while his wife is getting her infertility prescription refilled by the clinic pharmacist. As the nurse sits down beside him, he blurts, "It's like there are three of us in bed—my wife, me, and the doctor." What feeling is reflected by this statement?

Anger

After an abdominal hysterectomy the client returns to the unit with an indwelling catheter. The nurse notes that the urine in the client's collection bag has become increasingly sanguineous. What complication does the nurse suspect?

An incisional nick in the bladder

During a pelvic examination of a 24-year-old woman, the nurse suspects a vaginal infection because of the presence of a white curdlike vaginal discharge. What other assessment supports a fungal vaginal infection?

An itchy perineum -An itchy perineum usually occurs with candidiasis, a fungal infection; pruritus is the most common symptom

A client elects to have her pregnancy terminated after finding out at 16 weeks' gestation that she is carrying a fetus with Down syndrome. What should the nurse conclude about an abortion at this stage of the pregnancy?

An opportunity for the client to express feelings about her decision should be provided. -The client must feel comfortable enough to verbalize her feelings; this will help her complete the grieving process.

At 12 weeks' gestation, a client who is Rh negative expels the total products of conception. What is the nursing action after it has been determined that she has not been previously sensitized?

Administering RhoGAM within 72 hours

The clinic nurse is planning care for a client found to have chlamydia. Which treatment should the nurse plan to implement?

Administration of azithromycin (Zithromax) 1 g orally in a single dose

A couple in their late 30s, expecting their first child, plans to have an amniocentesis. At what point in the pregnancy should the nurse tell the couple that the test it will be scheduled?

After the 14th week of pregnancy

Twenty-four hours after an uncomplicated labor and birth a client's complete blood count reveals a white blood cell (WBC) count of 17,000/mm3. How should the nurse interpret this WBC count?

An expected response to the process of labor and birth

A client with a third-degree uterine prolapse is scheduled for a vaginoplasty. What should the nurse anticipate the surgeon will prescribe?

Apply moist compresses to the uterine prolapse. -Moist compresses may be prescribed to prevent ulcerations. Ambulation is contraindicated; it will predispose the client to the development of ulcerations. Elevating the foot of the bed is ineffective because gravity alone does not correct the prolapse. Supporting the prolapsed uterus with a sanitary pad may cause irritation and should be avoided.

One statement by a breastfeeding mother that indicates that the nurse's teaching about stimulating the let-down reflex has been successful is "I will:

Apply warm packs and massage my breasts before each feeding."

On the third postpartum day a client who is breastfeeding calls the clinic complaining of hot, hard, aching breasts. What recommendation should the nurse include in the response?

Apply warm, moist cloths to both breasts before nursing.

At her first prenatal clinic visit a primigravida has blood drawn for a rubella antibody screening test, and the results are positive. What intervention is important when the nurse discusses this finding with the client?

Asking her whether she has had German measles and when she had the disease -The positive result indicates that the client has had rubella or was vaccinated. The nurse should determine whether she has had the disease, because it is important to know whether it was before or after she became pregnant; if she had rubella at the start of her pregnancy, the fetus is at risk. A rubella booster, either at the next visit or after the birth, is not necessary because the client has active immunity. More information is needed before the client can be told that no treatment will be needed.

The nurse is teaching a client who is scheduling a vasectomy. What information is essential that the nurse explain to the client?

At least 15 ejaculations to clear the tract of sperm must occur before the semen is checked.

A client who is visiting the family planning clinic is prescribed an oral contraceptive. As part of teaching, the nurse plans to inform the client of the possibility of:

Breakthrough bleeding -Breakthrough bleeding, or midcycle bleeding, commonly occurs when women start using oral contraceptives. If it persists, the dosage should be changed. There is no evidence that cervicitis, ovarian cysts, or fibrocystic disease is related to the use of oral contraceptives.

A nurse is reviewing a postmenopausal client's history, which reveals that the client previously received hormonal replacement therapy (HRT) as treatment for osteoporosis. For which problem does HRT increase the client's risk?

Breast cancer

A nurse teaches a woman who is planning to breastfeed how to relieve breast engorgement. The nurse determines that further teaching is necessary when the woman states that she will:

Breastfeed the infant less frequently

How should a nurse prepare to administer Rho(D) immune globulin (RhoGAM) to a postpartum client?

By ensuring that the client is Rh negative and the neonate is Rh positive

A neonate is born with exstrophy of the bladder, and the parents are upset. They are told that corrective surgery will be performed as soon as possible. How can the nurse best help the parents at this time?

Caring for the newborn in the same manner as any other newborn

A primigravida at term is admitted to the birthing room in active labor. Later, when the client is dilated 8 cm, she tells the nurse that she has the urge to push. The nurse instructs her to pant-blow at this time because pushing can cause which of the following?

Cervical edema -The head cannot emerge when the cervix is not fully dilated. Pushing in this situation may cause cervical edema, predisposing the client to cervical laceration.

A couple interested in family planning ask the nurse about the cervical mucus method of family planning. The nurse explains that with this method the couple must avoid intercourse when and a few days after the cervical mucus is:

Clear and stretchable

The day after a client has a cesarean birth, the indwelling catheter is removed. The nurse concludes that urinary function has returned when the:

Client voids 300 mL of urine within 4 hours of catheter removal

A nurse in the postpartum unit must complete several interventions before a client's discharge from the hospital. The nurse plans to delegate some of the tasks to the nursing assistant. Which activity must be performed by the nurse?

Comparing the identification bands of mother and infant

What should the nurse teach a client about performing breast self-examination?

Compress the nipples to check for discharge.

After a modified radical mastectomy a client has two portable wound drainage systems in place. What is an important intervention as the nurse cares for these drainage systems?

Compressing the drainage receptacles after emptying them to maintain suction -Portable wound drainage systems are self-contained and may be emptied and compressed to reestablish negative pressure, which promotes drainage. Portable wound drainage systems are not irrigated; they drain by way of negative pressure. Portable wound drainage systems have collection chambers, so another drainage system is not needed. Portable wound drainage systems are self-contained closed systems.

A pregnant client tells the nurse that her husband is a chain smoker. What information should the nurse's teaching include?

Continued exposure to secondhand smoke is related to fetal growth restriction.

What should a nurse include in the discharge instructions for a woman who has undergone breast-conserving surgery (lumpectomy) for breast cancer?

Emphasizing the importance of breast self-examination -A client who has cancer of one breast is at risk for the development of cancer in the remaining breast; therefore breast self-examination is important.

A client with a history of endometriosis has abdominal surgery to remove adhesions. What should this client's postoperative plan of care include?

Encouraging the client to ambulate in the hallway -Muscle contraction during ambulation improves venous return, which prevents venous stasis and thrombus formation.

A nurse in the fertility clinic is instructing a client who will be using progesterone gel vaginally in the treatment of luteal phase infertility. When discussing the side effects of progesterone, what should the nurse tell the client to expect?

Enlarged, tender breasts

A client is taking a progesterone oral contraceptive (minipill). The nurse instructs the client to take one pill daily during the:

Entire menstrual cycle

A nurse discusses the type of anesthesia that will be used for a vaginal birth with a client who has class I cardiac disease. Which type of block is most appropriate for this client?

Epidural

What is the best nursing intervention to achieve the cooperation of an extremely anxious pregnant client during her first pelvic examination?

Explaining the procedure and maintaining eye contact while touching the client gently

A resident practitioner in the birthing unit asks the nurse to prepare for a vaginal examination on a client with a low-lying placenta who is in early labor. What is the priority nursing action?

Explaining why a vaginal examination should not be performed -A vaginal examination may cause separation of the placenta, resulting in hemorrhage. The nurse should discuss the situation with the resident, away from the client, because it is imperative that a vaginal examination not be performed without preparation for a cesarean birth.

A nurse in the fertility clinic works with couples who have been trying to become pregnant for more than 1 year. How can the nurse help ease the feeling of isolation that infertile couples often experience?

Explore ways to promote communication with family and friends

A couple in their late 30s who wish to have a child are referred for genetic counseling. They tell the nurse that they have a family history of an inheritable problem but have reservations about genetic counseling because they believe that genetic clinics favor abortion when the studies reveal a defective fetus. How should the nurse respond regarding genetic counseling?

Families are helped to understand the diagnosis, the probable cause of the disorder, and how the condition can be managed.

A client is admitted with a diagnosis of stage 0 cervical cancer (carcinoma in situ). What does the nurse emphasize while helping the client understand her diagnosis and prognosis?

Five-year survival rates for this cancer are nearly 100% with early treatment.

A nurse evaluates that a client who is taking oral contraceptives understands the related dietary teaching when the client states, "While I'm taking birth control pills I should increase my intake of foods containing:

Folic acid."

A client who is pregnant for the first time and carrying twins is scheduled for a cesarean birth. What should preoperative teaching include?

Frequent ambulation is begun within 24 hours.

A primigravida, unsure of the date of her last menstrual period, is told by the nurse that she appears to be at 22 weeks' gestation. What data support this conclusion?

Fundus at the umbilicus

After 2 weeks of radiation therapy for cancer of the breast a client experiences some erythema over the area being radiated. The area is sensitive but not painful. She states that she has been using tepid water and a soft washcloth when cleansing the area and applying an ice pack three times a day. What does the nurse conclude from this information?

Further teaching on skin care is necessary.

A nurse is obtaining a health history from a primigravida on her first visit to the prenatal clinic. Before discussing the client's health habits with her, what does the nurse consider the most important factor in the survival of the client's newborn?

Gestational age and birthweight

The nurse is caring for a group of postpartum clients. Which factor puts a client at increased risk for postpartum hemorrhage?

Giving birth to a baby weighing 9 lb 8 oz

The nurse is teaching a prenatal breathing and relaxation class. What does the nurse suggest to best ease back discomfort during labor?

Having support persons use back massage techniques -The fetus exerts pressure against the spine during labor; back massage provides counterpressure, which eases the discomfort.

A nurse determines that a client who, although ambivalent, is considering an abortion because of financial difficulties and is in crisis. How should the nurse intervene to alleviate the crisis?

Helping the client express her feelings

During the fourth stage of labor, the assessment of a primipara who has had a vaginal birth reveals a moderate to large amount of lochia rubra, a firm fundus that is at the umbilicus and deviated to the right, and pain that she rates as a 3 on a scale of 1 to 10. What is the priority nursing action?

Helping the client void -A fundus that is deviated to the right during the fourth stage of labor commonly is caused by a distended bladder ; if the bladder remains distended, involution will be inhibited, resulting in a boggy uterus that is prone to hemorrhage. The fundus is firm and does not need to be massaged. The fundus is firm; there is no need to increase the rate of the oxytocin (Pitocin) infusion. Because the client's pain is minimal, the priority is emptying the bladder to prevent hemorrhage.

A primipara delivered 12 hours ago. Although an ice bag has been applied to her perineal area, the client continues to complain of rectal pressure resulting in excruciating pain in the area of the episiotomy that is not relieved with analgesics. What does the nurse conclude is the cause of the client's pain?

Hematoma in the perineal area -Pain becomes excruciating with hematoma development at the episiotomy site because of pressure on surrounding nerve endings. This pain is not relieved by the application of ice because ice only reduces edema formation around the incision.

A client presents to the clinic with complaints of nausea and amenorrhea and reports that she obtained a positive result on a home pregnancy test. Which component of the history is most indicative of pregnancy?

Her urine immunoassay test is positive,

A 31-year-old client is seeking contraceptive information. Before responding to the client's questions about contraceptives, the nurse obtains a health history. What factor in the client's history indicates to the nurse that oral contraceptives are contraindicated?

History of borderline hypertension -Oral contraceptives may cause or exacerbate hypertension; borderline hypertension places the client at risk for a brain attack

A nurse is caring for four clients, each with a different medical condition. Which condition should the nurse anticipate will result in the client's being instructed by the health care provider not to breastfeed?

Human immunodeficiency virus -Breastfeeding by a mother infected with HIV is contraindicated because breast milk can transmit the virus to the infant

A nurse is caring for four clients, each with a different medical condition. Which condition should the nurse anticipate will result in the client's being instructed by the health care provider not to breastfeed?

Human immunodeficiency virus -Breastfeeding by a mother infected with HIV is contraindicated because breast milk can transmit the virus to the infant. Breastfeeding by a mother with mastitis is not always contraindicated; during antibiotic treatment the mother can maintain lactation by pumping the breasts and discarding the milk. Once the infection has resolved, breastfeeding may be resumed.

A client measuring at 18 weeks' gestation visits the prenatal clinic stating that she is still very nauseated and vomits frequently. Physical examination reveals a brown vaginal discharge and a blood pressure of 148/90 mm Hg. What condition does the nurse suspect the client is experiencing?

Hydatidiform mole -A hydatidiform mole, in which chorionic villi degenerate into grapelike vesicles, is the cause of these signs and symptoms. Although vomiting may cause dehydration, this conclusion ignores the vaginal discharge and hypertension.

A nurse is interviewing a female client with a tentative diagnosis of cystitis pending laboratory results. The nurse anticipates that the causative agent of the cystitis is Escherichia coli. The nurse anticipates this microorganism because it:

Inhabits the intestinal tract

A 16-year-old client has a steady boyfriend with whom she is having sexual relations. She asks the nurse how she can protect herself from contracting HIV. What should the nurse advise her to do?

Insist that her partner use a condom when having sex

A thin older adult client is found to have osteoporosis. What should the nurse include in the discharge plan for this client?

Instructions relative to diet and exercise

A pregnant woman asks the nurse when she may expect her baby to be born. She tells the nurse that her last menstrual period began on April 14. According to Nägele's rule, what is the client's expected date of birth (EDB)?

January 21

A 23-year-old woman arrives at the prenatal clinic because she thinks that she is pregnant. Her last menstrual period began on March 31, and her pregnancy test reveals a positive result. According to Nägele's rule, what is this client's expected date of birth (EDB)?

January 7

A client has a modified radical mastectomy because of a malignant tumor of the breast. What does the nurse plan to teach the client during the early postoperative period?

Keep the arm in an elevated position.

What nursing action best promotes parent-infant attachment behaviors?

Keeping the new family together immediately after the birth -Research strongly supports the theory that there is a sensitive period during the first few hours of life that is important for the promotion of parent-infant attachment. Parent-infant bonding can take place with or without visitors. Encouraging rooming-in is helpful because it increases the amount of contact between the parents and the newborn, but it is not as significant as those critical first few hours after the birth.

A woman is admitted for a hysterectomy and bilateral salpingo-oophorectomy. The nurse reviews the client's gynecological history. What condition does the client have that causes the nurse to anticipate an abdominal, rather than a vaginal, hysterectomy?

Large uterine fibroids -Attempting to remove a uterus with large uterine fibroids vaginally can cause trauma, resulting in hemorrhage. Vaginal hysterectomy is indicated for prolapsed uterus because the uterus is usually collapsed into the vagina. A hysterectomy is not the treatment of choice for mild cervical dysplasia; when a hysterectomy is necessary, the vaginal route is preferred. Urinary incontinence with coughing may be related to stress incontinence, which does not require a hysterectomy.

When seeing her preterm infant son in the neonatal intensive care unit for the first time, a mother exclaims, "He's so little! How will I ever be able to take care of him?" The nurse explains to the mother that she:

Will be encouraged to participate in his care as much as possible

A client in preterm labor is to receive a tocolytic medication, and bedrest is prescribed. Which position should the nurse suggest that the client maintain while on bedrest?

Lateral -The lateral position relieves pressure on the vena cava, thereby promoting venous return and increasing placental perfusion. The supine position promotes hypotension because the pressure of the gravid uterus on the vena cava interferes with the return of blood from the lower extremities. The Fowler position promotes hypotension because the pressure of the gravid uterus on the vena cava interferes with the return of blood from the lower extremities. The semi-Fowler position promotes hypotension because the pressure of the gravid uterus on the vena cava interferes with the return of blood from the lower extremities.

A client with cervical cancer is to undergo a course of internal radiation. The client returns to her lead-lined room on the oncology unit with an indwelling urinary catheter and a vaginal applicator in place. Once the practitioner has loaded the applicator with the radiation source, the nurse's plan of care should include:

Leaving the urinary catheter undisturbed

A nurse is caring for a client in labor whose cervix is dilated 6 cm. The client is receiving epidural analgesia. What common response to regional anesthesia does the nurse anticipate?

Lightheadedness

During a childbirth class the nurse determines that the women understand how to use effleurage correctly when they are observed:

Massaging their abdomens gently with their fingertips -Effleurage is a gentle massage of the abdomen that is effective during the first stage of labor because it distracts the client from the discomfort of the contractions. Rocking gently on the knees, known as the pelvic rock, is used during pregnancy to relieve backache. Practicing panting to avoid pushing during labor is a technique of breathing. Taking deep breaths before imagined contractions is also a technique of breathing.

The day after a hysterectomy, the client asks for sanitary pads because she feels that she is going to menstruate. What information should influence the nurse's response?

Menstruation will not occur because the uterus has been removed. -Menstruation is shedding of the endometrial lining of the uterus. A woman who has undergone a hysterectomy has had her uterus removed and will no longer menstruate. Abdominal pain is common after a hysterectomy, but menstruation is impossible after the surgery; additional symptoms are necessary before any conclusion can be made. Frank bleeding is not expected after a hysterectomy.

A nurse in the family planning clinic reviews the health history of a sexually active 16-year-old girl whose chief concern is a thick, burning discharge accompanied by a burning sensation and lower abdominal pain. After an examination the girl is informed that she may have a sexually transmitted infection (STI) that requires treatment. The adolescent is concerned that her parents will discover that she has been sexually active and asks the nurse whether her parents will be contacted. The nurse explains that her parents will:

Not be contacted, because treatment at the clinic is confidential

What should be included in nursing care immediately after a sexual assault?

Obtaining the assault history from the client

During a routine prenatal office visit at 26 weeks' gestation, a client states that she is getting fat all over and that she even needed to buy bigger shoes. What is the next nursing action?

Obtaining the client's weight and blood pressure

At a client's first visit to the prenatal clinic, the nurse asks the client when she had her last menstrual period so the estimated date of birth (EDB) can be determined. The client responds, "January 21." According to Nägele's rule, what is the month and day of the client's EDB?

October 28

A 38-year-old client attends the prenatal clinic for the first time. A nurse explains that several tests will be performed, one of which is the serum alpha-fetoprotein test. The client asks what the test will reveal. What should the nurse include in the reply?

Open neural tube defects

A female client with Hodgkin's disease is to start chemotherapy. She and her husband have been trying to have a child and are quite concerned when they learn that sterility may result. On what information should the nurse base the reply?

Ova can be harvested and frozen for future use.

A 37-year-old client with endometriosis visits the women's health clinic because she has dysmenorrhea and dyspareunia. What is a description of dysmenorrhea?

Pain with menses

The nurse presents a program on breast self-examination. After a return demonstration the nurse concludes that she needs to review certain aspects of the teaching program. Which behavior by one of the students supports this conclusion?

Palpating each breast while in the sitting position

A mother is concerned that her newborn will be exposed to communicable diseases when she goes home. While teaching the mother ways to decrease the risk of Infection; what type of immunity should the nurse explain was transferred to her baby through the placenta?

Passive natural -Passive natural immunity is developed from an antigen-antibody response in the mother that is transmitted to the fetus. Active natural immunity is acquired by an individual in response to a disease or an infection. Active artificial immunity is acquired by an individual in response to small amounts of antigenic material (e.g., vaccination). Passive artificial immunity is conferred by the injection of antibodies prepared in another host.

A histogram (hysterosalpingography [HSG]) is performed to determine whether a client has a tubal obstruction. The nurse explains to the client that infertility caused by a defect in the tube is most often related to:

Past infection

The nurse is caring for a client who is in the taking-in phase of the postpartum period. The area of health teaching that the client will be most responsive to is:

Perineal care

What is the focus of the nurse's anticipatory guidance during the first trimester of pregnancy?

Physical changes of pregnancy

A left modified radical mastectomy is performed on a client with breast cancer. What is the most important measure to be included in the care plan for the first postoperative day?

Placing the client in the semi-Fowler position with the left arm and hand elevated

A strict vegetarian (vegan) becomes pregnant and asks the nurse whether there is anything special she should do in regard to her diet during pregnancy. What is most the important measure for the nurse to instruct the client to take?

Plan to eat from specific groups of vegetable proteins each day.

A woman comes into the clinic and states that she is thinking about becoming pregnant. What can the woman do to improve the health of her baby before she becomes pregnant?

Start taking prenatal vitamins -Folic acid is important for the pregnant woman; a lack of folic acid can result in neural tube defects, including spina bifida. The time during fetal development when this occurs is very early in the pregnancy, when the woman may not even realize that she is pregnant. Taking prenatal vitamins with adequate folic acid can greatly reduce this birth defect.

A client at 40 weeks' gestation is admitted to the birthing unit in labor. During the initial examination the nurse uses Leopold maneuvers to palpate the abdomen. The purpose of this intervention is to assess the:

Position of the fetus -Duration of contractions is timed by lightly placing the examining hand on the fundus during a contraction or observing the electronic monitor. The station can be ascertained during a vaginal examination, which is not part of the Leopold maneuvers. The nurse palpates the abdomen to locate the head, back, and small parts of the fetus; the locations of these parts reveal the position of the fetus. Frequency of contractions is timed by lightly placing the examining hand on the fundus for several minutes or by observing the electronic monitor.

A nurse is teaching a breastfeeding client about medications that are safe and unsafe for her to take. Which medication is contraindicated?

Propylthiouracil (PTU) -The concentration of propylthiouracil (PTU) excreted in breast milk is three to 12 times higher than its level in maternal serum; this may cause agranulocytosis or goiter in the infant. Heparin (Hep-Lock) is not excreted in breast milk. The amount of breast milk excretion of gentamicin (Garamycin) is unknown, but it can be given to infants directly without adverse effects. Diphenhydramine (Benadryl) is excreted in breast milk, but it does not adversely affect the infant when therapeutic doses are given to the mother.

A nurse is teaching a class of expectant parents about nutritional needs during pregnancy. What information should the nurse include?

Protein needs increase to at least 70 g/day during pregnancy.

A nurse is discussing immunizations needed to confer active immunity with a pregnant client during her first visit to the prenatal clinic. What information should the nurse consider including that the client will understand with regard to active immunity?

Protein substances are formed by the body to destroy or neutralize antigens.

A client in preterm labor is receiving subcutaneous terbutaline (Brethine) tocolytic therapy. The nursing action that is most important during the initial administration of this medication is assessing the client's:

Pulse rate continuously -Tachycardia is an expected side effect of terbutaline , a betamimetic agent. The pulse rate should be monitored continuously, and the rate should be no more than 120 beats/min. The reflexes are not affected by terbutaline, which relaxes uterine musculature. This medication does not affect the client's level of consciousness. Although intake and output are assessed, an indwelling catheter is not indicated.

A pregnant woman tells a nurse, "I think I can feel the baby move now. It feels like butterflies in my stomach. My friend calls it feeling life." What term should the nurse include when discussing fetal movement with the woman?

Quickening

A nurse is counseling a pregnant client who is a vegetarian. What should the nurse plan to do to ensure optimal nutrition during the pregnancy?

Refer the client to a dietitian to help plan her daily menu

A client who recently gave birth is transferred to the postpartum unit by the nurse. What must the nurse do first to avoid a charge of abandonment?

Report the client's condition to the responsible staff member

A nurse discusses herpes genitalis as part of a high school sex education program. The nurse explains to the students that herpes genitalis is:

Responsible for local as well as systemic reactions

A nurse is preparing a client with a ruptured tubal pregnancy for immediate surgery. What type of surgery should the informed consent include?

Salpingectomy

A client states that she wishes to use the calendar method of birth control. The nurse concludes that the client understands how to calculate the beginning of the fertile period when she states, "I will:

Subtract 18 days from the length of my shortest cycle" -The fertile period is determined by subtracting 18 days from the length of the shortest cycle to determine the first unsafe day and subtracting 11 days from the length of the longest cycle to determine the last unsafe day.

A nurse is planning to teach a new mother about breastfeeding. What should the nurse consider before preparing the client to breastfeed?

Suckling stimulates the release of oxytocin.

A primipara about to be discharged with her newborn asks the nurse many questions regarding infant care. What phase of maternal adjustment does this behavior illustrate?

Taking-hold -The taking-hold phase, which begins about the second or third postpartum day, involves concern about being a "good" mother; the new mother is most receptive to teaching at this time. Let-down is not related to bonding. The let-down reflex refers to the flow of milk in response to suckling and is caused by the release of oxytocin from the posterior pituitary. The taking-in phase is the first period of adjustment to parenthood. It includes the first 2 postpartum days; the mother is passive and dependent and preoccupied with her own needs. The behavior described refers to the taking-hold phase of bonding. Early parenting involves many behaviors, of which taking hold is only one.

After an emergency cesarean birth, the client tells the nurse that she was hoping for a "natural" childbirth but is glad that she and her baby are all right." Which postpartum phase of adjustment does this statement most closely typify?

Taking-in -By discussing the experience, the client is bringing it into reality; this is characteristic of the taking-in phase. The client is not ready to assume the tasks of the letting-go phase until the tasks of the taking-in and taking-hold phases have been completed. The taking-hold phase is marked by an increased desire to resume independence. The working-through phase is not a separate phase of adjustment to parenthood; this is not relevant.

A nurse is teaching a childbirth preparation class about the discomfort of labor. What is the greatest influence on the perception of pain for a woman in labor?

Tension of the client

A client is admitted to the birthing unit because fluid is leaking from her vagina. She is unsure whether her "bag of water" has broken. What should the nurse do to help determine whether the fluid is amniotic fluid?

Test the fluid with nitrazine paper -Amniotic fluid is slightly alkaline, and urine is acidic; when moistened with amniotic fluid, nitrazine will turn dark blue, indicating an alkaline substance. Inspecting the fluid is a subjective assessment and may be inaccurate.

A young sexually active client at the family planning clinic is advised to have a Papanicolaou (Pap) smear. She has never had a Pap smear before. What should the nurse include in the explanation of this procedure?

The Pap smear can detect cancer of the cervix.

A pregnant client with diabetes is referred to the dietitian in the prenatal clinic for nutritional assessment and counseling. What should the nurse emphasize when reinforcing the client's dietary program?

The need to eat a pregnancy diet that meets increased dietary needs and to adjust the insulin dosage as necessary

Laboratory studies reveal that a pregnant client's blood type is O, and she is Rh positive. The client asks whether her newborn will have a problem with blood incompatibility. Before responding, the nurse must remember that fetal problems may develop if the fetus is:

Type A or B

A nurse is caring for a client who is being given intravenous magnesium sulfate to treat preeclampsia. Which adverse side effect alerts the nurse to notify the health care provider?

Urine output of less than 100 mL in 4 hours

A woman questions the nurse about the effectiveness of oral contraceptives. What most important factor about the effectiveness of oral contraceptives should be included in the reply to this question?

User motivation -Conception will not be prevented unless the user is motivated to use the method correctly and consistently.

A young client tells the nurse that her mother complains about having dysmenorrhea and asks the nurse what this means. How should the nurse describe dysmenorrhea?

Uterine pain during the menstrual period

A nurse at a women's health clinic confirms that client teaching regarding the use of an oral contraceptive is understood when the client states, "I:

Will put a baby's picture on my bathroom mirror so I'll see it every morning" -Putting a baby's picture on the bathroom mirror serves as a reminder that the oral contraceptive must be taken every day. A woman should wait 2 to 3 months after stopping the oral contraceptive pill before attempting to become pregnant. If two consecutive menstrual cycles are missed, the client should stop the contraceptive pill and perform a pregnancy test. The client should use a barrier method of contraception for the first month of pill use and when a pill is missed to help prevent conception.


संबंधित स्टडी सेट्स

Abeka History 5 ch.7 (pg.105-113)

View Set

Advanced Cyber: Ch 13: Vulnerability Assessment & Data Security

View Set

Medical Ethics Final Chapters 1-13

View Set

Potter & Perry Ch 32 - Medication Administration (Practice Questions)

View Set

Econ 101 Chapter 6 Supply Demand and Government Policies

View Set

AP World Unit 2 Practice Questions

View Set

Chapter 20: Blood Vessels and Circulation

View Set

Slave Narrative-Honors English (Enderby/Hilty)

View Set